Law Of Contract Quiz: Multiple Choice Questions!

Approved & Edited by ProProfs Editorial Team
The editorial team at ProProfs Quizzes consists of a select group of subject experts, trivia writers, and quiz masters who have authored over 10,000 quizzes taken by more than 100 million users. This team includes our in-house seasoned quiz moderators and subject matter experts. Our editorial experts, spread across the world, are rigorously trained using our comprehensive guidelines to ensure that you receive the highest quality quizzes.
Learn about Our Editorial Process
| By Dmatski
D
Dmatski
Community Contributor
Quizzes Created: 1 | Total Attempts: 5,839
Questions: 63 | Attempts: 5,846

SettingsSettingsSettings
Law Of Contract Quiz: Multiple Choice Questions! - Quiz

Are you studying law? Are you familiar with the law of contract? Then take this law of contract quiz that consists of multiple-choice questions (MCQs) with answers. Contract law is the form of law that pertains to the creation and enforcement of pacts. A contract is an arrangement that a party can go to court to protect. Contract law is the part of the law that governs creating contracts. When you are in a business, you generally utilize contract law. This quiz will explain the law of contract. All the best.


Questions and Answers
  • 1. 

    Integrated Circuit Technology, Inc. ('ICT') was having difficulty finding qualified engineers to work in its expanding production facilities because the hardware needed to train top-flight engineering students was so expensive that most colleges could not afford to purchase more than a minimal amount of such equipment, which limited program sizes and held down the number of applicable engineering degree candidates. ICT's chief executive officer, Cruz, wrote the following letter to the heads of several universities:  If you will commit your institution to accepting in your engineering degree program every junior-year undergraduate who wishes to pursue that major, ICT will provide your institution with all the necessary electronic hardware to instruct such students. Our engineers have informed me that one set of equipment (i.e., one each of every machine used by ICT in its production process) is sufficient for use by five students; therefore, for every five engineering degree majors you accept in excess of your current engineering degree population, ICT will provide one such set of equipment. Jackson, chancellor of Seaport College of Engineering ('SCE'), also received a letter from Cruz. The ICT letter arrived during SCE's annual budget review process, and Jackson was called upon to decide how much of the college's limited funds to apply to engineering equipment purchases. The day that she received Cruz's letter, Jackson informed the budget committee that no funds would be needed from that year's budget for engineering equipment purchases. A few days later, as Jackson was dictating the acceptance letter to ICT, she received a second letter from Cruz indicating that the ICT equipment program was being canceled due to an influx of qualified foreign engineers to the United States. Because the budget had been approved and all available funds had been committed to nonengineering equipment purchases, Jackson mailed the acceptance letter anyway and it was delivered to Cruz at ICT. If SCE brings a breach of contract action against ICT, what result?

    • A.

      Judgment for SCE, if Jackson's removal of funds from the engineering department's budget was a reasonable and foreseeable response to ICT's offer.

    • B.

      Judgment for SCE, because Jackson's removal of funds from the engineering department's budget converted ICT's offer into an express option.

    • C.

      Judgment for ICT, because its promise to supply equipment to SCE was not supported by consideration.

    • D.

      Not recover, because he was in a drunken stupor when injured.

    • E.

      Judgment for ICT, because its revocation was received by Jackson before she dispatched the acceptance letter.

    Correct Answer
    A. Judgment for SCE, if Jackson's removal of funds from the engineering department's budget was a reasonable and foreseeable response to ICT's offer.
    Explanation
    (A) Under the modern, majority view, where the offeror could reasonably expect that the offer would induce the offeree to detrimentally change position in reliance on the offer, the courts will hold the offer to be an irrevocable option due to the detrimental reliance. (D) would be the correct answer under the older, minority view, which did not recognize the application of promissory estoppel in this situation, and would thus apply the rule that the revocation, effective on receipt, predated dispatch of the acceptance. (B) is wrong because the offer is converted into an equitable option, not an express option. (C) is incorrect because SCE's agreement to expand its degree program and accept applying students is adequate consideration to support ICT's promise.

    Rate this question:

  • 2. 

    In July of last summer George, a grape grower, contracted with Walter's Winery to deliver '500 tons of premium quality pinot chardonnay grapes grown on my ranch Grapeacre in Grape County.' The price was to be $1,000 per ton and delivery was to be on or before September 15. In August of the same year, George entered into an identical contract with Vinnie's Vintner Co. to sell 300 tons of premium quality pinot chardonnay grapes. George completed his harvest by September 10 and had 800 tons of premium quality grapes. On September 11, an unexpected rain ruined 400 tons, and George notified Walter and Vinnie on that day that he would only be able to deliver 250 tons to Walter and 150 tons to Vinnie. On September 14, Vinnie purchased an additional 150 tons of premium quality pinot chardonnay grapes from Godfrey, one of several other available sources for premium quality pinot chardonnay grapes. These grapes along with the 150 tons from George gave Vinnie the 300 tons he needed. On September 15, what is Walter's Winery's legal position with regard to George's failure to deliver the 500 tons of grapes required by his contract?

    • A.

      If Walter has given George a written notice of termination, Walter will have the right to refuse to accept the 250 tons of grapes but will have no cause of action for damages against George.

    • B.

      Even if Walter has given George a written notice of termination, Walter must accept the 250 tons of grapes and will have no cause of action for damages against George.

    • C.

      Since Vinnie's purchase establishes that it is possible for George to perform by obtaining additional grapes from other available sources, Walter may accept the 250 tons from George and recover damages for George's failure to deliver the balance of the amount specified by the contract.

    • D.

      Since George's contract with Walter was entered into before his contract with Vinnie, George is bound to deliver the entirety of his grape crop to Walter.

    Correct Answer
    A. If Walter has given George a written notice of termination, Walter will have the right to refuse to accept the 250 tons of grapes but will have no cause of action for damages against George.
    Explanation
    (A) This is a question requiring precise knowledge of the U.C.C. The problem is governed by U.C.C. sections 2-615 and 2-616. A crop failure resulting from an unexpected cause excuses a farmer's obligation to deliver the full amount as long as he makes a fair and reasonable allocation among his buyers. George has done this by allocating pro rata between Walter and Vinnie. Nevertheless, under U.C.C. section 2-616 the buyer may either accept the proposed modification or terminate the contract. Thus,
    (B) is wrong.
    (C) is wrong because even though alternative sources are available, George is not obligated to use them because the contract was tied to a designated parcel of land|'my [George's] ranch.'
    (D) is wrong because it is contrary to the provision of U.C.C. section 2-615, which permits the farmer to make an allocation.

    Rate this question:

  • 3. 

    Christine and her friend Zelda were going away to college and had quite a few personal belongings to transport. Christine's father offered to drive them in his van, but when the van was loaded they discovered there was only room for two people, so Christine asked her boyfriend Harry to drive her in his car while Christine's father and Zelda rode in the van. About halfway to the college, while the van and Harry's car were driving down the freeway, the van in front, the van suddenly swerved out of control and ran off the highway, ending up on its side in the center divider. When Harry stopped his car and Christine ran to the van, she discovered to her horror that her father was dead. Zelda appeared to be injured, but not severely. Because her father previously had heart trouble, Christine assumed that he had had a heart attack while driving, although a later investigation would reveal that the accident was caused solely by a defect in the steering mechanism of the van. Filled with remorse, Christine told Zelda, 'I'm so sorry about this. I'll make good any losses you suffer because of this accident.' Later, when Christine learned that Zelda was going to seek treatment from Dr. Winston, she wrote the doctor a letter stating that she would be responsible for all of Zelda's medical expenses; Dr. Winston received the letter the next day. Assume for purposes of this question only that several months after the accident, but within the applicable statute of limitations, Zelda discovered that she had suffered an injury to her spinal column that would prevent her from ever playing basketball again. Zelda had been a scholarship athlete in basketball at the college and was considered to be a certain high draft selection for the newly formed women's professional basketball league when she graduated. She brought an action against Christine for several million dollars in damages. Which of the following is the best defense Christine could assert against Zelda's claim?

    • A.

      There was no consideration supporting her promise to Zelda to make good any losses.

    • B.

      She did not intend to offer to pay Zelda for the loss of her professional career when she said she would make good any losses.

    • C.

      She was in error when she assumed that her father's heart attack was the cause of the accident.

    • D.

      She did not know that Zelda would not be able to play basketball when she offered to make good any losses.

    Correct Answer
    A. There was no consideration supporting her promise to Zelda to make good any losses.
    Explanation
    (A) Zelda did not promise to do anything she was not otherwise obligated to do in bargained-for exchange for Christine's promise, so no consideration supported the latter's promise. If the agreement were otherwise enforceable, none of the other facts offered by the answers would prevent it from being carried out.
    (B) is incorrect because a court will not examine the promisor's subjective intent when making a promise.
    (C) is incorrect because mistake as to a fact that motivated the contract is not a defense under these circumstances; Christine had assumed the risk of mistake by making the promise without being certain of the cause of the accident.
    (D) is similarly incorrect. Mistake as to the value of the promise is generally a risk assumed by the parties.

    Rate this question:

  • 4. 

    On January 2, Smith borrowed $1,000 from his friend Jones, agreeing in writing to repay the loan within a year. In September, it became clear to Smith that he would have difficulty meeting the year deadline, and so Smith approached Brown with the following proposition: Smith would perform 200 hours of work for Brown during the next six months at the special rate of $5 per hour, if Brown would agree to pay $1,000 for the entire 200 hours to Jones on the following January 1. Brown agreed. By January 1, Smith had only worked five hours for Brown, and Brown stated to Smith that he would not pay Jones because Smith had not worked enough. Smith responded, 'That's okay, just hold on to the money until I get 200 hours in, then pay Jones.' Brown agreed. Assume for purposes of this question that after the agreement between Smith and Brown was entered into in September, Smith informed Jones of the arrangement. Jones's response was, 'I don't care who pays me but if I don't get my money by January 2, I'll sue.' The January 1 modification between Smith and Brown occurs, and after learning of the modification, Jones sues Brown for $1,000. What is the probable result of this action?

    • A.

      Judgment for Jones, because he was informed of the original agreement and did not participate in the modification.

    • B.

      Judgment for Jones, because he assented to the original agreement.

    • C.

      Judgment for Brown, because the original agreement was modified before Jones's rights became vested.

    • D.

      Judgment for Brown, because his contract was with Smith, not Jones.

    Correct Answer
    B. Judgment for Jones, because he assented to the original agreement.
    Explanation
    (B) A modification of the contract can take place without the consent of the third-party beneficiary prior to the time the third-party beneficiary's rights become vested. The rights become vested when the third party assents in a manner requested by the parties, detrimentally relies on the contract, or brings a lawsuit to enforce it. Here, the parties asked Jones to assent and, although his answer may have been grumbling, he assented. Since Jones's rights were then vested, the contract could not subsequently have been modified without his consent. Thus,

    (C) and

    (D) are wrong.

    (A) does not go far enough; mere knowledge of the original arrangement is not enough to vest rights.

    Rate this question:

  • 5. 

    Tortfeasor tortiously injured Victim in an auto accident. While Victim was recovering in Hospital, Tortfeasor's liability insurer, Insurer, settled with Victim for $5,000. Victim gave Insurer a signed release and received a signed memorandum wherein Insurer promised to pay Victim $5,000 by check within 30 days. When Victim left Hospital two days later, Hospital demanded payment of its $4,000 stated bill. Victim thereupon gave Hospital his own negotiable promissory note for $4,000, payable to Hospital's order in 30 days, and also, as security, assigned to Hospital the Insurer settlement memorandum. Hospital promptly assigned for value the settlement memorandum and negotiated the note to Holder, who took the note as a holder in due course. Subsequently, Victim misrepresented to Insurer that he had lost the settlement memorandum and needed another. Insurer issued another memorandum identical to the first, and Victim assigned it to ABC Furniture to secure a $5,000 credit sale contract. ABC immediately notified Insurer of this assignment. Later it was discovered that Hospital had mistakenly overbilled Victim by the amount of $1,000 and that Tortfeasor was an irresponsible minor. In view of Tortfeasor's age and irresponsibility when Insurer issued his liability policy, can Holder and ABC Furniture recover on their assignments?

    • A.

      Neither can recover because Victim, the assignor, is a third-party beneficiary of the liability policy, whose rights thereon can be no better than Tortfeasor's.

    • B.

      Neither can recover unless Insurer knowingly waived the defense of Tortfeasor's minority and irresponsibility.

    • C.

      Neither can recover because the liability policy, and settlement thereunder, are unenforceable because of Tortfeasor's minority.

    • D.

      Either Holder or ABC Furniture, depending on priority, can recover as assignee (or subassignee) of Victim's claim because the latter arose from Insurer's settlement agreement, the latter agreement not being vitiated by Tortfeasor's minority and irresponsibility when he obtained the policy.

    Correct Answer
    D. Either Holder or ABC Furniture, depending on priority, can recover as assignee (or subassignee) of Victim's claim because the latter arose from Insurer's settlement agreement, the latter agreement not being vitiated by Tortfeasor's minority and irresponsibility when he obtained the policy.
    Explanation
    (D) Insurance contracts usually cannot be voided by infants. Thus, the liability insurance, and settlement agreement arising out of it, are valid.

    (A) seems to be based on the incorrect assumption that, because of Tortfeasor's minority at the time the liability policy was issued, Insurer might not be bound on the contract of insurance. Actually, the effect of a contract entered into between an infant and an adult is that the contract is voidable by the infant but binding on the adult. Thus, there is no question that Tortfeasor has rights under the liability policy.

    (B) assumes that Tortfeasor's minority constitutes a defense. As explained above, this is incorrect.

    (C) is incorrect for the same reason.

    Rate this question:

  • 6. 

    QUESTIONMater, a wealthy widow, wishing to make a substantial and potentially enduring gift to her beloved adult stepson, Prodigal, established with the Vault Savings and Loan Association a passbook savings account by an initial deposit of 10000.For this question only, assume the following facts. The passbook was issued solely in Prodigal's name; but Mater retained possession of it, and Prodigal was not then informed of the savings account. Subsequently, Mater became disgusted with Prodigal's behavior and decided to give the same savings account solely to her beloved adult daughter Distaff. As permitted by the rules of Vault Savings and Loan, Mater effected this change by agreement with Vault. This time she left possession of the passbook with Vault. Shortly thereafter, Prodigal learned of the original savings account in his name and the subsequent switch to Distaff's name. If Prodigal now sues Vault Savings and Loan for $10,000 plus accrued interest, will the action succeed? 

    • A.

      Yes, because Prodigal was a thirdparty intended beneficiary of the original Mater-Vault deposit agreement.

    • B.

      Yes, because Prodigal was a constructive assignee of Mater's claim, as depositor, to the savings account.

    • C.

      No, because Prodigal never obtained possession of the passbook.

    • D.

      No, because Prodigal's right, if any, to the funds on deposit was effectively abrogated by the second Mater-Vault deposit agreement.

    Correct Answer
    D. No, because Prodigal's right, if any, to the funds on deposit was effectively abrogated by the second Mater-Vault deposit agreement.
    Explanation
    (D) Prodigal was a third-party beneficiary of the original Mater-Vault deposit agreement. A thirdparty beneficiary's rights do not vest
    (i.e., are modifiable) until he: manifests assent to the promise, brings suit to enforce the promise, or materially changes position in justifiable reliance on the promise. None of these vesting acts occurred before the modification here; in fact, Prodigal did not even know of the first contract until after the modification. Thus, his rights had not vested, and the terms of the second Mater-Vault deposit agreement control the disposition of the funds. Thus,

    (A) is incorrect.

    (B) is incorrect, because even if Prodigal were an assignee, this gratuitous assignment would be revocable.

    (C) is incorrect because Prodigal's rights could have vested without obtaining possession of the passbook.

    Rate this question:

  • 7. 

    In early January 2004, representatives of MacDougall Corporation, makers of the famous MacDougall Dog hot dog and related convenience foods sold through thousands of owned and franchised 'MacDougall's' restaurants, met with representatives of Time Management, Inc. ('TM'), a firm specializing in time-and-motion studies of labor intensive industries. After extensive negotiations, it was orally agreed that TM would redesign the food production area of MacDougall's restaurants, including modification of cooking equipment, if necessary, so that, using existing MacDougall's food products, savings in labor costs through reduction in restaurant cooking staffs would result. Lawyers for MacDougall's subsequently drafted a written agreement, sent it to TM, whose lawyers modified the draft, and returned the modified draft to MacDougall's. This modified writing, signed by both parties, stated in its entirety: Provided that at least 2,000 work-hours per restaurant are eliminated, MacDougall Corporation will pay to TM within 90 days of installation of new food production systems at MacDougall's restaurants in Richmond a first installment of $1 million. Upon installation of new food processing systems nationwide, MacDougall Corporation will pay to TM a second and final installment of $1.5 million. Nationwide installation must be completed by January 15, 2005. Any amendments to this agreement must be in writing signed by both parties. TM immediately began work on the restructuring of MacDougall's food processing methods. On September 5, 2004, a radical change in the layout of MacDougall's kitchen area and new personnel assignments had been designed, and TM demanded payment of the first installment payment of $1 million. MacDougall Corporation refused, but negotiations conducted between the parties resulted in an oral agreement that MacDougall's would pay $750,000 immediately and then the $1.5 million second installment as originally agreed, after nationwide installation of the new system. The restructured food production system was installed and in operation in all Richmond MacDougall's restaurants on October 1, 2004. Subsequent audits revealed that the new system enabled MacDougall Corporation to eliminate 1,500 work-hours per restaurant, saving the corporation $90,000 in labor costs for all Richmond restaurants. The new system required that MacDougall's increase the length of the famous MacDougall Dog by three centimeters and that the 'Mother MacDougall Hot Apple Fritters' be made in a rectangular shape rather than the traditional round form. Nationwide installation of the new system in all MacDougall's restaurants was completed on January 30, 2005. The 1,500 work-hours per restaurant savings to MacDougall Corporation was projected at $1.8 million per year. TM sent a certified letter to the chief executive officer of MacDougall Corporation requesting his certification that the new food production system was in place and operating as promised, and demanding the $1.5 million second installment. The CEO refused to so certify and refused to make any payment, noting in his reply letter that the system had not been installed by January 15, 2005, and that it did not use existing MacDougall's food products, as promised by TM. Assume for the purpose of this question only that an express condition of Mac- Dougall Corporation's duty to pay the contract price failed and that TM was in breach because it failed to complete nationwide installation of the food processing system by January 15, 2005. If TM brings an action to recover the reasonable value of its services, will it likely succeed?

    • A.

      No, because failure of an express condition precedent would excuse MacDougall Corporation of its duty to pay TM.

    • B.

      No, because a claim for reasonable value of services would be inconsistent with a claim by MacDougall Corporation against TM for breach of contract.

    • C.

      Yes, because MacDougall Corporation continued to use the new food processing system and was aware that TM expected to be paid for its services.

    • D.

      Yes, because MacDougall Corporation continued to use the new food processing system and would realize $1.8 million per year as a consequence of the contractual relationship between the parties.

    Correct Answer
    D. Yes, because MacDougall Corporation continued to use the new food processing system and would realize $1.8 million per year as a consequence of the contractual relationship between the parties.
    Explanation
    (D) TM will likely succeed because MacDougall will otherwise be unjustly enriched by getting the system without having to pay the second installment. Where quasi-contractual relief is used to remedy a failed or unenforceable contract, all that is necessary is that the failed contract result in unjust enrichment of one of the parties. Even a party who has breached a contract
    (and therefore cannot enforce it) can recover in quasi-contract under the modern view as long as the breach did not involve seriously wrongful or unconscionable conduct. Hence, neither the failure of the express condition
    (A) nor the breach of contract claim
    (B) would preclude TM from recovering in quasi-contract for the reasonable value of its services.
    (C) is not as good a choice as

    (D) because all that is necessary to recover in a failed contract situation is that the other party would otherwise be unjustly enriched. Where there is no contractual relationship between the parties, the party seeking quasi-contractual relief must specifically show that his expectation of being compensated is reasonable and that the benefits were conferred at the express or implied request of the defendant, but those elements are assumed when the unjust enrichment occurs in a failed contract situation. Hence,

    (D) states the rule in this situation more accurately than
    (C).

    Rate this question:

  • 8. 

    Producer hired Fiddler to play in an orchestra that was to leave on a 10-week tour of the United States. Fiddler, a musician, turned down another job opportunity in order to accept Producer's job offer. One week after the start of the tour, Fiddler was hospitalized with a bad back and was unable to perform. Producer hired Player to take Fiddler's part in the orchestra. Four days later, Fiddler recovered but Producer refused to allow Fiddler to rejoin the orchestra or to complete the tour. Fiddler then sued Producer for breach of contract. Which of the following is Fiddler's best legal theory?

    • A.

      His reliance on the job offered by Producer by declining another job opportunity created an estoppel against Producer.

    • B.

      His failure to perform with the orchestra for four days was not a material failure so as to discharge Producer's duty to perform.

    • C.

      His performance with the orchestra for the four-day period was physically impossible.

    • D.

      Fiddler was never told that an injury might jeopardize his continued employment with the orchestra.

    Correct Answer
    B. His failure to perform with the orchestra for four days was not a material failure so as to discharge Producer's duty to perform.
    Explanation
    (B) The contract extended for a 10-week period. Missing only four days of a performance to run for 10 weeks would not be considered a total material breach of the contract.

    (A) is wrong because this would tend to go more to a formation problem. Here the contract has already been validly formed.

    (C) is wrong because a claim of impossibility could conceivably discharge the entire contract, and, therefore, Fiddler would have no claim.

    (D) is wrong because Fiddler need not be warned of the consequences of a breach.

    Rate this question:

  • 9. 

    Aunt Sheila told her niece, Sinead, 'I'd like you to go to Tara Imports and select the $300 lace shawl of your choice and I'll buy it for you if you wear it to the ethnic festival. I want you to look as sweet and delicate as an Irish rose.' Sinead, a modern 22-year-old, despised shawls because she thought they were 'the sorts of things old ladies wear.' Also, her taste in music ran to heavy metal rock and roll, rather than the traditional Celtic bagpipe and fiddle music she would be subjected to if she went to the ethnic festival. However, Sinead really loved Aunt Sheila and did not want to hurt her feelings. She went to Tara Imports and purchased a $300 lace shawl imported from Ireland. Sinead accompanied Sheila to the ethnic festival wearing the shawl, and Sheila was very pleased. Assume for purposes of this question only that Sheila died shortly after the festival and her estate refuses to reimburse Sinead for her purchase. Sinead filed suit. Her attorney advanced four legal theories on which he asserts that Sinead can collect the $300 from Sheila's estate. They are as follows: (I. Promissory estoppel. (II. Bargain and exchange. (III. Conditional gift. (IV. Account stated. (Which of the following represents the correct combination of legal theories that support Sinead's case against Sheila's estate?

    • A.

      I. and II.

    • B.

      I. and III.

    • C.

      II. and IV.

    • D.

      I., II., III., and IV.

    Correct Answer
    A. I. and II.
    Explanation
    (A) Sinead can use both a promissory estoppel theory and a bargain and exchange theory to collect from Sheila's estate. To be enforceable, a contract must be supported by consideration or there must be a substitute for consideration. Generally, consideration requires a bargained-for exchange of something of legal value from each party. To be bargained-for, the promise must induce the detriment and the detriment must induce the promise. Here, Sinead was induced to purchase the shawl and incur a debt in exchange for Aunt Sheila's promise to pay, and Aunt Sheila was induced to pay in exchange for Sinead's promise to purchase the shawl and attend the festival. It does not matter that Aunt Sheila did not receive an economic benefit|influencing Sinead's mind is sufficient to establish an exchange. Both parties gave something of legal value since Aunt Sheila had no legal duty to pay for a shawl for Sinead and Sinead had no legal duty to purchase a shawl and attend the festival. Therefore, there was a bargained-for exchange of something of legal value and a contract was formed. Thus, II. states a theory that is helpful to Sinead. Promissory estoppel
    (I.) is also helpful. Promissory estoppel is a substitute for consideration: A promise is enforceable to the extent necessary to prevent injustice if the promisor should reasonably expect to induce action of a definite and substantial kind, and such action is induced. Here, Aunt Sheila's promise would reasonably induce Sinead to buy the shawl and she was in fact so induced. Thus, promissory estoppel would allow Sinead to recover. Neither of the other two theories will lead to enforcement of Aunt Sheila's promise. A promise to make a gift is unenforceable for lack of consideration, and this result is not changed where the promise to make the gift is conditional and the conditions have been fulfilled. In fact, an argument that there was a conditional gift would negate the bargain and exchange needed to establish an enforceable contract, since it would involve arguing that Aunt Sheila did not intend to induce Sinead's detriment (i.e., Aunt Sheila was not seeking to trade her promise to pay in exchange for Sinead's promise to attend), but rather was merely stating upon what conditions Sinead could receive her gift. Therefore, III. is not helpful to Sinead. An argument that there was an account stated
    (IV.) is inappropriate because that is a contract where parties who have had a series of transactions agree to a final balance due from one to the other. Here, Sinead is seeking to enforce a single transaction. Therefore,

    (A) is correct, and

    (B),

    (C), and

    (D) are wrong.

    Rate this question:

  • 10. 

    Civil service rules, which have been on the books in the city of Charlesville for many years, provide that any member of the police department must serve a one-year probationary period before he or she will be considered a permanent employee. In fact, this rule was enacted before Charlesville had a police academy, and now a prospective police officer spends six months in the academy before being hired by the city. Ruby, a graduate of the police academy, was with the city police department for eight months when she was terminated. There were no city ordinances or state laws that required that Ruby be given a reason for the termination or a hearing, and she was given neither. Which of the following facts, if shown, gives the city of Charlesville the strongest argument for refusing to give Ruby a statement of reasons why her employment was terminated and for denying her the opportunity to contest the termination?

    • A.

      Ruby, as a female, did not perform as a police officer as well as her male counterparts.

    • B.

      Ruby had failed to include in her application the fact that during college she was a member of a radical student organization.

    • C.

      Ruby had not been granted permanent employment status.

    • D.

      Ruby had graduated in last place in her class at the police academy.

    Correct Answer
    C. Ruby had not been granted permanent employment status.
    Explanation
    (C) While on probation, Ruby was subject to being fired at any time. Thus, she had no right in the nature of a property right and, constitutionally, would not be entitled to a hearing.
    (A),
    (B), and
    (D) are all arguable reasons for terminating Ruby's employment. However, just because the city might have had a valid reason for firing her, it does not follow that, if she were otherwise entitled to a hearing, the city could forgo it.
    (A), in addition, is not a good argument if the standard by which Ruby was tested was an artificial standard based solely on the physical differences between men and women, having no rational relationship to the needs of the job of a police officer.

    Rate this question:

  • 11. 

    Jenny, a general contractor, advertised in a trade publication that she planned to bid on the construction of a new building to be located in the Civic Mall. The advertisement welcomed bids from subcontractors to perform various functions, such as plumbing, electrical work, and masonry. The lowest plumbing bid was from Plunger, who bid $10,000. Jenny used Plunger's bid in preparing her general bid. At 2 p.m. on June 22, Jenny submitted her general bid. At 3 p.m. Plunger called her and said, 'I'm sorry, Jenny, but I made a mistake on that bid I submitted to you; I can't possibly do that plumbing work for a dime less than $12,000.' Jenny told him, Look, you've done a lot of good work for me in the past and we all make mistakes. I'll just forget you ever made that $10,000 bid. Plunger effusively thanked Jenny. Jenny then hired Flusher to do the plumbing work for $12,000. She then sued Plunger for damages. Jenny will:

    • A.

      Win, because there was no additional consideration to support a release.

    • B.

      Win, because the dollar amount of the agreement is large enough that the Statute of Frauds applies.

    • C.

      Lose, because a rescission has taken place.

    • D.

      Lose, because Jenny and Plunger mutually agreed to a release.

    Correct Answer
    C. Lose, because a rescission has taken place.
    Explanation
    (C) The unilateral option contract between Plunger and Jenny to keep Plunger's offer open was effectively rescinded by Jenny's expressed intent to make a gift of the obligation owed her. The typical case of rescission involves a bilateral contract where neither party has yet performed; i.e., the duties of both parties are still executory. However, no contract to do the plumbing work has been created yet, because Jenny has not communicated her acceptance of the bid to Plunger. Despite her use of Plunger's bid to prepare her own bid, she is free to award the plumbing work to someone else if she is awarded the general contract. Hence, the contract to do the plumbing cannot be rescinded because it has not been created. Another contract is present under this fact pattern, however. Under section 87 of the Restatement
    (Second) of Contracts, Plunger's offer is binding as an option contract because Jenny reasonably relied on it to submit her bid. The option contract here is unilateral: Jenny's acceptance of the option contract by using the bid also constituted performance of her duties under the option contract. In a unilateral contract case, a rescission promise must be supported by either
    (i) an offer of new consideration,
    (ii) elements of promissory estoppel
    (i.e., detrimental reliance), or
    (iii) the offeree's manifestation of an intent to make a gift of the obligation owed her. The first two alternatives are absent in these facts, but the gift alternative is indicated by Jenny's statement that she will 'forget' that Plunger ever made the bid. Jenny's response was an effective rescission of the option contract.

    (A) is an incorrect choice even though it is a true statement. A discharge of contractual duties by means of a release requires additional consideration or some substitute, such as a signed writing or reliance by the offeror on the discharge. Here there is no additional consideration to support a release, as choice
    (A) indicates, but Jenny will not win because a rescission has taken place.

    (B) is incorrect because a large dollar amount for purposes of the Statute of Frauds is irrelevant unless there is a contract for the sale of goods, which must be in writing if the goods are priced at $500 or more. The agreement between Jenny and Plunger involved a contract for services, which is not within the $500 provision of the Statute.

    (D) is incorrect because, as discussed above, a release requires additional consideration, a signed writing, or detrimental reliance by the offeror. Since none of these is indicated by the facts, a release has not taken place.

    Rate this question:

  • 12. 

    Zeller contracted in writing to deliver to Baker 100 bushels of wheat on August 1 at $3.50 per bushel. Because his suppliers had not delivered enough wheat to him by that time, Zeller on August 1 had only 95 bushels of wheat with which to fulfill his contract with Baker. If Zeller tenders 95 bushels of wheat to Baker on August 1, and Baker refuses to accept or pay for any of the wheat, which of the following best states the legal relationship between Zeller and Baker?

    • A.

      Zeller has a cause of action against Baker, because Zeller has substantially performed his contract.

    • B.

      Zeller is excused from performing his contract because of impossibility of performance

    • C.

      Baker has a cause of action against Zeller for Zeller's failure to deliver 100 bushels of wheat.

    • D.

      Baker is obligated to give Zeller a reasonable time to attempt to obtain the other five bushels of wheat.

    Correct Answer
    C. Baker has a cause of action against Zeller for Zeller's failure to deliver 100 bushels of wheat.
    Explanation
    (C) Since Baker contracted for 100 bushels on August 1, Zeller had a contractual duty which was breached by the tendering of only 95 bushels. Baker has a cause of action for that breach.

    (A) is wrong because substantial performance does not excuse total performance of contractual duties.
    (B) is wrong because discharge by impossibility only occurs where impossibility is objective. The fact that Zeller only had 95 bushels created a subjective rather than an objective impossibility of performance, and thus did not excuse his breach.

    (D) is wrong because Baker was entitled to performance on August 1.

    Rate this question:

  • 13. 

    Wendy, a wealthy widow, owned a prime piece of land in an exurban area populated by affluent residents. Wendy had a daughter, Dorothea, who was Wendy's only child and the 'apple of her eye.' Dorothea was 23 years of age and engaged to be married to Pemberton d'Argent, a rich, polo-playing investment banker. Wendy wanted to give Dorothea a very special wedding gift. Wendy therefore entered into a written agreement with contractor Brikk whereby Brikk would build a house on the property for $300,000. The house was to be built to very exacting specifications that described in great detail the materials to be used, the exact shape of each room, etc. These specifications were included in the written agreement between Wendy and Brikk. The agreement provided that Wendy would pay Brikk $300,000 upon completion of the building according to specifications and that Brikk would turn the keys to the home over to Dorothea. After the agreement was signed by both Wendy and Brikk, Dorothea and Pemberton looked at some fine homes in the same general area. They had two opportunities to purchase suitable homes at good prices, but decided to turn them down. Since then, property values in the area have increased by approximately 30%. Just as Brikk was about to begin construction of the house, he discovered that an underground river bisected Wendy's property. This left insufficient subterranean support to construct the house as planned. Assume for purposes of this question only that upon discovering the underground river, Brikk refused to try to build the house for $300,000. If Wendy files suit demanding specific performance or damages from Brikk, which of the following additional facts, if proven, would most favor Wendy's case?

    • A.

      It is physically possible to build the house according to the original specifications by the sinking of many pilings deep into the ground for support, although it would add $1 million to Brikk's costs.

    • B.

      The detailed specifications in the agreement had been drawn up by Brikk, as were other blueprints and plans for the house.

    • C.

      Neither Wendy nor Brikk had reason to know of the underground river before the contract was signed.

    • D.

      Dorothea knew of the contract between Wendy and Brikk, and her sole reason for turning down the opportunities to purchase suitable housing was reliance on the contract.

    Correct Answer
    B. The detailed specifications in the agreement had been drawn up by Brikk, as were other blueprints and plans for the house.
    Explanation
    (B) Wendy's strongest argument is that Brikk had drawn up both the detailed specifications of the agreement and the plans for the house. When both parties entering into a contract are mistaken about facts relating to the agreement, the contract may be voidable by the adversely affected party if (i) the mistake concerns a basic assumption on which the contract is made;
    (ii) the mistake has a material effect on the agreed-upon exchange; and
    (iii) the party seeking avoidance did not assume the risk of the mistake. Here, it is likely that neither party knew that the soil conditions were unsuitable. But if Brikk had drawn up the specifications and plans for the house, it will be more likely that the court will conclude that he has assumed the risk of any mistake as to the sufficiency of the soil to support the house, and will not be able to avoid the contract.

    (A) might be helpful to Wendy, but it might also be harmful to her and so is not as good an answer as

    (B). The fact that it is physically possible to build the house at an increased cost negates the defense of impossibility
    (which is also unavailable because the impossibility existed at the time the contract was entered into). However, the argument may aid a defense of impracticability, which is available where the party to perform has encountered extreme and unreasonable difficulty in performing which was not anticipated by either party. While a moderate increase in the cost of building would not be sufficient for impracticability, the fact that performance here will cost over 300% more than the anticipated cost will support the impracticability defense. [See Restatement (Second) of Contracts |261, comment d]

    (C) would not be as helpful to Wendy's case as

    (A) because it only indicates that there was a mutual mistake, and a contract can be avoided for mutual mistake unless the adversely affected party bears the risk of the mistake, which the fact in
    (B) would establish.

    (D) is irrelevant because whether Dorothea, the third-party beneficiary, knew of the contract affects only her ability to enforce the contract. Wendy, as a party to the contract, may enforce regardless of whether Dorothea knew of it.

    Rate this question:

  • 14. 

    On February 1, Ridewell Rubber Co. telephoned Smithson Tire Shop and offered to sell to Smithson 500 series 4 Ridewell tires for $20,000. Smithson accepted immediately. On February 3, Smithson sent Ridewell a letter confirming the deal and stating that Smithson was counting on a 20% discount due to the size of the purchase. On February 20, Ridewell telephoned Smithson and stated that it could not afford to sell the 500 series 4 tires for less than 30000. If Smithson brings suit against Ridewell and Ridewell asserts the Statute of Frauds as a defense, will Smithson prevail?

    • A.

      Yes, but only if its February 3 letter contained the quantity term.

    • B.

      Yes, regardless of whether its February 3 letter contained the quantity term because the letter merely confirms a prior deal for 500 series 4 tires.

    • C.

      No, because Smithson's February 3 letter varied the terms of Ridewell's offer.

    • D.

      No, because Ridewell is the party to be charged and has signed nothing.

    Correct Answer
    A. Yes, but only if its February 3 letter contained the quantity term.
    Explanation
    (A) If the quantity was stated in the February 3 letter, the Statute of Frauds is satisfied and Smithson may prevail. This contract is for the purchase and sale of goods; thus, the U.C.C. applies. The Statute of Frauds requires that a contract for the sale of goods for $500 or more must be in writing. However, U.C.C. section 2-201(2) provides that in a deal between merchants, a writing confirming the deal sent by one party will bind both parties, unless the other party objects in writing within 10 days. Also, the U.C.C. requires that the quantity term be included in the writing for it to be sufficient under the Statute of Frauds. [U.C.C. |2-201(1)] Here, Ridewell's objection came after 10 days and so Smithson's letter confirming the deal is sufficient for the Statute of Frauds if it lists the quantity.

    (B) is wrong because the quantity term is the key to the sufficiency of a memorandum. All other terms may be proved by parol evidence, but the quantity term must be in the writing.

    (C) is wrong because it does not bear on the Statute of Frauds issue, but rather on the issue of the additional terms.

    (D) is wrong because of the rule stated above that in a contract between merchants a written confirmation by one party binds both parties, unless objected to within 10 days. Thus, Ridewell can be charged although it has not signed the memorandum.

    Rate this question:

  • 15. 

    Integrated Circuit Technology, Inc. ('ICT') was having difficulty finding qualified engineers to work in its expanding production facilities because the hardware needed to train top-flight engineering students was so expensive that most colleges could not afford to purchase more than a minimal amount of such equipment, which limited program sizes and held down the number of applicable engineering degree candidates. ICT's chief executive officer, Cruz, wrote the following letter to the heads of several universities: If you will commit your institution to accepting in your engineering degree program every junior-year undergraduate who wishes to pursue that major, ICT will provide your institution with all the necessary electronic hardware to instruct such students. Our engineers have informed me that one set of equipment (i.e., one each of every machine used by ICT in its production process) is sufficient for use by five students; therefore, for every five engineering degree majors you accept in excess of your current engineering degree population, ICT will provide one such set of equipment. Marchand, chancellor of Middle States University ('MSU'), received a letter from Cruz and immediately mailed a reply letter accepting ICT's generous offer. A few weeks later, Marchand received another letter from Cruz explaining that the response to his original letter had been so overwhelming that schools such as MSU who had no history of supplying graduates to ICT as employees would have to be omitted from ICT's equipment program. If MSU brings an action for breach of contract against ICT, what result?

    • A.

      Judgment for ICT, because it could not be determined with sufficient specificity what MSU's needs for equipment would be, there having been no time to ascertain how many new engineering degree students would apply to the expanding degree program.

    • B.

      Judgment for ICT, because MSU's obligation under any contract is illusory|no additional students will necessarily apply to an expanded engineering degree program.

    • C.

      Judgment for MSU, because ICT is classified as a merchant under the U.C.C. and its offer was therefore irrevocable.

    • D.

      Judgment for MSU, because Marchand's letter was an effective acceptance of ICT's offer and an enforceable contract was thereby formed.

    Correct Answer
    D. Judgment for MSU, because Marchand's letter was an effective acceptance of ICT's offer and an enforceable contract was thereby formed.
    Explanation
    (D) Marchand's letter of acceptance created an enforceable contract.

    (A) is wrong because requirements contracts are now recognized as sufficiently definite to be enforced.

    (B) is incorrect because MSU's promise was not illusory; it agreed to expand its degree program and to accept anyone who applied.

    (C) is not the best answer because ICT's offer was accepted before the purported revocation, and so the revocability of the offer is immaterial.

    Rate this question:

  • 16. 

    Martha needed a new pair of shoes. She went to her local Skysheim shop and told the salesperson that she worked in the city's downtown area and had to walk eight blocks to get from her house to her commuter train and then six blocks from her train to her office. During the workday, she had to climb up and down stairs several times. She wanted shoes that were suitable for walking on concrete, had gripping power for stairs, and were comfortable. The salesperson went into his stockroom and brought out four different styles of Skysheim's 'Clouds,' reputably the most comfortable shoe on the market, designed for the type of use that Martha had in mind. Martha tried on each of the four pairs but did not like the way any of them looked. While walking around the store, however, Martha saw a shoe she did like|'Hobblers,' Skysheim's high-fashion shoe. She told the salesperson to bring her a pair to try. He did so and explained to Martha that Hobblers were completely made of the finest leather and would probably last for years. Martha tried on the shoes and told the salesperson that she would take them. Assume for this question only that Martha bought the shoes and wore them twice. She decided that they were too uncomfortable for her daily commute. She took the shoes back to the Skysheim shop and demanded her money back. Skysheim refused. If Martha sues to get her money back, under which theory would she most likely prevail? I. Breach of the implied warranty of fitness for particular purpose. II. Breach of the implied warranty of merchantability.III. Breach of express warranty.

    • A.

      I. and II., but not III.

    • B.

      I. and III., but not II.

    • C.

      II. and III., but not I.

    • D.

      None of the above.

    Correct Answer
    D. None of the above.
    Explanation
    (D) An implied warranty of fitness for particular purpose arises where a seller knows that the buyer is relying on the seller to pick suitable goods for the buyer and the buyer relies on the seller's expertise. Here, Martha asked the seller to pick suitable goods, but declined the seller's advice and picked out her own shoes. Therefore, this warranty did not arise. The implied warranty of merchantability arose but was not broken. This warranty arises in every sale of goods by a merchant and ensures, generally, that goods shall be fit for ordinary purposes. Nothing in the facts indicates that the shoes here are not fit for ordinary purposes. The only express warranties here arose from the statement that the shoes were leather and, perhaps, from the sample shoe on the sales floor. However, nothing in the facts indicates that there has been a breach of either of these express warranties. Therefore,

    (D) is the correct answer.

    Rate this question:

  • 17. 

    Hair of the Dog, a small pet store, entered into a written contract with Pet Products, Inc. whereby Pet Products agreed to supply Hair of the Dog with whatever quantity of pet food it might order, at a mutually agreed price, for two years, with an option to renew. Also, the contract required that Hair of the Dog buy its pet food from Pet Products only. For the first six months of the contract, Hair of the Dog ordered from Pet Products 10 cases of pet food per month. In the seventh month, the owner of Hair of the Dog sold the shop, inventory, and accounts receivable to Amalgamated Pet Shops, a chain operation. As part of the sale, Hair of the Dog assigned to Amalgamated the contract with Pet Products. Amalgamated promptly notified Pet Products of the sale and assignment. That same month, Amalgamated, looking to stock the pet food in stores throughout its chain, sent Pet Products an order for 5,000 cases of pet food on the terms and conditions of your agreement with Hair of the Dog, which has been assigned to us. Pet Products did not have the means to fill such a large order and refused to deliver 5,000 cases. If Amalgamated brings suit, the court should hold that:

    • A.

      Amalgamated is entitled to enforce the agreement, as it gave Pet Products prompt notice of the assignment from Hair of the Dog.

    • B.

      There was no mutuality of obligation in the original agreement between Pet Products and Hair of the Dog and hence there was nothing to 'assign' to Amalgamated.

    • C.

      Amalgamated cannot compel Pet Products to fill the order for 5,000 cases of pet food.

    • D.

      Amalgamated is entitled to enforce the agreement if it is willing to pay cash, as one person's credit is not necessarily as good as another's.

    Correct Answer
    C. Amalgamated cannot compel Pet Products to fill the order for 5,000 cases of pet food.
    Explanation
    (C) Amalgamated cannot compel Pet Products to fill the order. An assignment of rights is barred if it will substantially change the obligor's duty. At common law, the right to receive goods under a requirements contract generally was not assignable because the assignment could change the obligation of the parties. Under the U.C.C., such a right might be assignable if the quantity requirement is not unreasonably disproportionate to the quantity originally contemplated by the parties. [U.C.C. |2-306] Here, the owner of Hair of the Dog owned only one store and his requirements for the first six months of the contract were only 10 cases per month. Amalgamated, on the other hand, is a chain and its requirements are 5,000 cases per month, a disproportionately large increase. Therefore, the assignment here would be unenforceable even under the Code.

    (A) is incorrect because prompt notice of the assignment does not obviate the problem of the unreasonably disproportionate requirements.

    (B) is incorrect because there is mutuality of obligation
    (consideration on both sides) here. The parties entered into a requirements contract whereby Pet Products agreed to sell Hair of the Dog all of its pet food requirements and Hair of the Dog promised to buy its pet food only from Pet Products.
    (Hair of the Dog's promise to buy only what it 'might order' is not illusory because it agreed to not purchase pet food from any other source.) A requirements contract is not illusory because the U.C.C. imposes a duty to purchase requirements in good faith.

    (D) is incorrect because it implies that the assignment here is not assignable because the assignee poses a different credit risk from the assignor
    (and the risk can be eliminated through cash payment). However, as discussed above, the contract here is not assignable because of the unreasonably disproportionate requirements of Amalgamated, not because of Amalgamated's creditworthiness or lack thereof.

    Rate this question:

  • 18. 

    Babe was a professional baseball player who was known both for his prowess at the plate and his perceived 'heart of gold.' One day, Babe was visiting a sick boy named Jimmy in the hospital. Babe was touched by Jimmy's will to live despite a very poor prognosis. In a moment of weakness, Babe told Jimmy that in consideration of Jimmy's courage, he would do anything that Jimmy asked. Jimmy's eyes momentarily gleamed as he asked Babe to 'hit a homer for me in your next game.' Babe replied, 'Sure kid.' As Babe was leaving Jimmy's hospital room, Jimmy's father, Joe, pulled Babe aside and told Babe, 'It would mean a lot to Jimmy if you would hit a home run for him in your next game. The medicinal value of raising Jimmy's spirits would be priceless.' Babe replied, 'Hey man, we all have problems. I don't work for the Make a Wish Foundation.' Undaunted, Joe repeated that it would really raise Jimmy's spirits if Babe would hit a homer, and as incentive, Joe told Babe that he would pay Babe $5,000 if Babe did hit a home run in his next game. Babe replied, 'You've got a deal.' To raise his chances of collecting the $5,000 from Joe, Babe took extra batting practice before his next game, and the practice paid off because in his next game, Babe hit two home runs. During a post-game interview, Babe explained, 'I did it for little Jimmy, who is in the hospital.' After showering, Babe went directly to Joe's house and asked Joe for $5,000. Babe's contract with his ball club does not forbid him from accepting money from fans for good performance. If Joe refuses to pay and Babe brings an action against Joe for damages, which of the following is correct under the modern trend in contract law?

    • A.

      Babe can recover the $5,000 because the preexisting duty rule does not apply where the duty is owed to a third person.

    • B.

      Babe can recover the $5,000 if he can prove that the value of the home run to Jimmy is at least $5,000.

    • C.

      Babe cannot recover from Joe because Babe had a preexisting duty to use his best efforts to hit home runs.

    • D.

      Babe cannot recover from Joe because, even under the modern trend, moral consideration is not valid.

    Correct Answer
    A. Babe can recover the $5,000 because the preexisting duty rule does not apply where the duty is owed to a third person.
    Explanation
    EXPLANATION
    (A) Babe can recover because, under the modern trend, the preexisting duty rule does not apply if the duty is owed to a third person. Generally, contracts must be supported by consideration. A promise to perform is valid consideration, but if a person already owes a duty to perform, traditionally that performance cannot be used as consideration for another promise. Thus, under the traditional rule, Babe could not enforce Joe's promise to pay Babe $5,000 if Babe hit a home run because Babe gave no valid consideration in exchange for Joe's promise, since Babe owed a preexisting duty to his ball club to exert his best efforts to hit home runs. However, under the modern view as formulated in Restatement
    (Second) of Contracts, section 73, a duty is a preexisting duty only if it is owed to the promisor. Thus, a promise to perform a duty is valid consideration as long as the duty of performance is not already owed to the promisee. In other words, if the duty is owed to a third party, a promise to perform given to another is valid consideration as long as it was bargained for.

    (B) is incorrect because there is no exception to the preexisting duty rule|modern or otherwise|that allows the promisor to recover merely because his performance benefited the third party. Babe can recover under the modern approach because his promise to Jimmy's father was bargained for. Conversely, Babe does not have to prove that the value of his home run to Jimmy was at least $5,000, because courts generally will not inquire into the adequacy of consideration.
    (C) would be correct under the traditional rule, but this question is one of the few questions where you are asked about and expected to know the modern trend. As explained above, under the modern trend, the promise here is valid consideration because the duty to hit home runs was owed to a third party
    (the ball club) rather than to the promisee
    (Joe).

    (D) is wrong because while it is true that moral consideration is not good consideration, Joe did not rely on moral consideration, but rather exchanged a promise to pay $5,000 for Babe's performance.

    Rate this question:

  • 19. 

    Patrick was exasperated with the smog in Big City and sent Andrew the following letter on January 1: Andrew, my family and I are moving out of here and going to live on a tropical island. Do you want to buy the stuff in our house? The price is 25000 Andrew received the letter on January 2, and on January 3 sent Patrick a letter accepting the offer. The next day Andrew changed his mind. He called Patrick and told him to forget the deal. Later that day, Patrick received the letter that Andrew had sent on January 3. Is there a contract between Patrick and Andrew?

    • A.

      Yes, because the contract is for the sale of goods for more than $500 and Patrick's attempted rejection is oral.

    • B.

      Yes, because Andrew's letter of acceptance was effective when he mailed it.

    • C.

      No, because Andrew's rejection was communicated to Patrick before his letter of acceptance was received.

    • D.

      No, because the description of the subject matter as 'the stuff in our house' is not sufficiently definite and certain.

    Correct Answer
    B. Yes, because Andrew's letter of acceptance was effective when he mailed it.
    Explanation
    (B) Andrew accepted Patrick's offer when he mailed his letter on January 3, and thus, a contract was formed. Under the 'mailbox' rule, acceptance of an offer by mail creates a contract at the moment the acceptance is posted properly stamped and addressed. If the offeree sends both an acceptance and a rejection, whether the mailbox rule will apply depends on which the offeree sent first, the acceptance or the rejection. If the offeree first sends an acceptance and later sends his rejection, the mailbox rule does apply. Thus, even if the rejection arrives first, the acceptance is effective upon mailing
    (and so a contract is formed) unless the offeror changes his position in reliance on the rejection. Here, Andrew first sent an acceptance, then called with his rejection. The mailbox rule applies, and since there is nothing in the facts to show that Patrick relied on the rejection, a contract was formed.

    (A) is wrong because it implies that a rejection must be in writing. There is no such requirement. Also, the rejection
    (absent detrimental reliance) has no effect on the contract since the offer had already been accepted and the contract formed.

    (C) is wrong because, as stated above, under the mailbox rule the fact that the rejection was received before the acceptance is irrelevant
    (unless there has been detrimental reliance on the rejection, which was not the case here). The contract was formed when Andrew sent his acceptance.

    (D) is wrong because the description, although somewhat ambiguous, can be made reasonably certain by evidence of the subjective understanding of the parties and extrinsic evidence of what was in the house, which a court will consider to clarify an ambiguous term.

    Rate this question:

  • 20. 

    Lehman was a limited partner in Bountiful Homes, a partnership organized by Lehman's nephew, Sanders, which purchased land, subdivided it, then constructed and sold single-family residences on the lots thus created. Sanders was the sole general partner. During the construction of the only housing development undertaken by the partnership, Lehman discovered that Sanders had taken most of the money invested by the limited partners and all of the money paid to the partnership by purchasers of the homes in the development and lost it gambling in Las Vegas. When confronted by Lehman, Sanders admitted everything, then went to his apartment and committed suicide. When news of Sanders's suicide was made public, Lehman was besieged by creditors of the partnership and by people who had purchased homes. The jurisdiction statutorily limited the liability of limited partners for debts of the partnership or acts of the general partner to the extent of their investment in the partnership, but being unaware of this, Lehman believed himself liable to all who had claims against Bountiful Homes. He told Wolcott, a single mother whose house was partly completed, that he would make good any losses caused by my nephew's actions, and then orally agreed with Smith, a contractor, to pay for the completion of Wolcott's house. Lehman also told Brubaker, an unsecured creditor of Bountiful Homes, that if Brubaker would hold off filing an involuntary bankruptcy petition against the partnership, Lehman would pay the partnership debt. In a bankruptcy action filed by secured creditors of the partnership, the assets of the partnership, which were very small, were consumed by the costs of the proceedings and no creditor received any payment. Sanders himself left no assets and was in fact heavily indebted on a personal basis due to his compulsive gambling. If Smith did not complete construction of Wolcott's house and Wolcott brings an action against him for breach of contract, which of the following would be an effective defense for Smith? I. Smith's contract was with Lehman. II. Wolcott furnished no consideration. III. Any agreement between Wolcott and Smith was discharged by novation because of the agreement between Smith and Lehman.

    • A.

      I. only.

    • B.

      I. and II. only.

    • C.

      II. and III. only.

    • D.

      None of the above.

    Correct Answer
    D. None of the above.
    Explanation
    (D) None of the defenses would be effective for Smith. I. is not valid because it does not matter that Smith's contract was with Lehman; Wolcott was an intended third-party beneficiary of their contract. An intended beneficiary can enforce the contract once her rights have vested, such as by manifesting assent to the promise in a manner invited or requested by the parties. Here, Wolcott's rights would have vested when she assented to have Smith do the work. II. is not valid; the fact that Wolcott furnished no consideration is immaterial because consideration was furnished by Lehman. III. is not valid. A novation occurs when a new contract substitutes a new party to receive benefits and assume duties that had originally belonged to one of the original parties under the terms of the old contract. Here, the fact that Smith subsequently contracted with Lehman does not establish a novation; there was no agreement between all of the parties to discharge the existing contract. Thus,

    (D) is correct, and

    (A),

    (B), and

    (C) are incorrect.

    Rate this question:

  • 21. 

    Ben and Sandy, brother and sister, received a $50,000 inheritance from their deceased father. By mutual agreement, they used the money to purchase a 10-acre parcel of land. Ben and Sandy took title as joint tenants. Three years after the purchase, Ben suggested to Sandy that they build an apartment house on the property. Sandy rejected this idea. Ben then asked if he could build an apartment house on his half of the property; Sandy agreed. Ben then built an apartment house on the eastern five acres of the property. Six months later, Sandy gave permission to the Boy Scouts of America to use the western half of the property as a site for weekend camping trips. Two years later, Ben died, leaving his entire estate to his son, Steven. In an appropriate action to determine the respective interests of Sandy and Steven in the property, if Sandy is adjudged to be the owner of all of the property, the most likely reason for the judgment will be that:

    • A.

      The Statute of Frauds prevents the enforcement of Sandy's oral agreement.

    • B.

      The record title of the property as joint tenancy can be changed only by a duly recorded instrument.

    • C.

      Ben could not unilaterally sever the joint tenancy.

    • D.

      Ben's expenditure of funds in building the apartment house in reliance on Sandy's oral promise estops her from denying the oral permission.

    Correct Answer
    A. The Statute of Frauds prevents the enforcement of Sandy's oral agreement.
    Explanation
    (A) The Statute of Frauds prevents the enforcement of an oral agreement concerning an interest in land.
    (B) is wrong because there is no issue as to the record title to the land. The agreement between Ben and Sandy, if it had been reduced to writing, would have been perfectly valid as between them.
    (C) is wrong because a joint tenancy can be unilaterally severed.
    (D) is wrong because this would not support Sandy's ownership of the entire piece of land.

    Rate this question:

  • 22. 

    Sam was a famous auto racer and builder of racing cars. He and Bob signed a contract for sale of one of Sam's hand-built race cars for $25,000, the price to be paid and the car to be delivered one week later. The day after the contract was signed, Sam called Bob and told him that Sam's wife, Winnie, who had a half interest in the race car, would not go along with the sale at $25,000. Winnie would agree to a sale for $40,000. If Winnie in fact has a half interest in the racing car:

    • A.

      There is no enforceable contract because the car cannot be sold unless both owners convey title.

    • B.

      There is an enforceable contract only if Bob was unaware of Winnie's interest when he signed with Sam.

    • C.

      There is an enforceable contract regardless of whether Bob was aware of Winnie's interest at the time he signed.

    • D.

      The contract is discharged by prospective inability of performance.

    Correct Answer
    C. There is an enforceable contract regardless of whether Bob was aware of Winnie's interest at the time he signed.
    Explanation
    (C) Sam's unconditional promise to sell created a contract even if Bob knew of Winnie's interest. When a promise is unconditional, the failure to perform according to its terms is a breach of contract. By not making his promise conditional on Winnie's consent to convey her interest, Sam impliedly undertook to obtain her consent. Therefore, the contract is enforceable.
    (Note that this does not necessarily mean that Bob will be able to get the car; he may have to settle for damages because of Winnie's interest.)
    (A) is incorrect although partially true. It is true that Sam cannot sell Winnie's half of the car without her consent; however, that does not make the contract here unenforceable. As stated above, by making his promise unconditional, Sam undertook a duty to obtain Winnie's consent to sell the car. His failure to do so is a breach of contract, but a breach does not negate a contract; it merely gives the nonbreaching party a right to certain remedies. Therefore, the contract is enforceable, even if Winnie refuses to sell her interest.
    (B) is incorrect because Bob's knowledge of Winnie's interest is irrelevant to the issue of the contract's enforceability. As discussed above, Sam's unconditional promise implied that Sam would obtain Winnie's consent to convey her interest in the car. Therefore, the contract is enforceable regardless of whether Bob was aware of Winnie's interest at the time he signed.
    (D) is incorrect because prospective inability is not a ground for discharge. Prospective failure of consideration occurs when a party has reasonable grounds to believe that the other party will be unable or unwilling to perform when due. The prospective inability of performance does not discharge the contract; rather, it allows the innocent party to suspend further performance until he receives adequate assurances that performance will be forthcoming. Therefore, the contract between Sam and Bob is not discharged because of prospective inability of performance.

    Rate this question:

  • 23. 

    Odivia owned Homeacre, on which both a house and a garage were located. Odivia did not own an automobile, and she decided that she would turn the garage into an exercise area, including a modern sauna and spa. Odivia entered into a written agreement with contractor Eero, who agreed to do the job personally for $12,500, which included all requisite plumbing, electrical, and carpentry work. Eero was to begin work by May 14. On May 15 Eero had not yet appeared to start the job. Odivia telephoned Eero, who told her, 'I've got a big job with Developers Incorporated that's going to pay me a lot more money than that marginal project of yours, so I'm not going to work on your garage.' Over a period of several months, Odivia made many calls to local contractors, but none of them would agree to do the job for the price agreed upon by Eero. On June 3 of the following year Odivia filed suit for specific performance against Eero. Which of the following represents Eero's best argument in his defense against Odivia's suit?

    • A.

      Specific performance is an equitable remedy, and because Odivia waited for over a year to sue, the equitable defense of laches will apply.

    • B.

      Specific performance is inappropriate, because a contract for personal services is involved.

    • C.

      Specific performance is not an appropriate remedy if nominal legal damages are available to Odivia.

    • D.

      Specific performance is inappropriate, because Odivia's failure to obtain another contractor for the job is an indication that $12,500 was an unfair price.

    Correct Answer
    B. Specific performance is inappropriate, because a contract for personal services is involved.
    Explanation
    (B) Eero's best argument is that a personal services contract is not specifically enforceable. Thus, Odivia cannot obtain specific performance of Eero's agreement to perform personal plumbing, electrical, and carpentry work for her. One of the prerequisites to obtaining specific performance is that a plaintiff must show that the legal remedy is inadequate. Where the plaintiff has contracted for something rare or unique, money damages are inadequate compensation for loss of the bargain. Generally, services to be performed under a personal services contract are not unique and money damages can remedy a breach. Thus, specific performance is not available in such cases. In addition, even in the case of unique services, a court will not order a defendant to work for the plaintiff, in part because of the difficulty of enforcement and because such an order is tantamount to unconstitutional involuntary servitude. Another requirement for specific performance is that enforcement must be feasible. Enforcing a personal services contract generally would create complicated and time-consuming supervision problems that courts are reluctant to undertake. In this case, Eero agreed to personally perform for Odivia plumbing, electrical, and carpentry work. Thus, this was a personal services contract. However, the services to be performed by Eero were not unique or capable of being performed solely by Eero. Odivia could obtain adequate compensation by receiving the amount, above her contract price with Eero, that it will cost to have someone else perform the required work
    (plus reasonable compensation for the delay in performance). Thus, specific performance is inappropriate here.

    (A) is incorrect because circumstances that would permit the defense of laches do not appear to be present. Laches is available as an equitable defense if the plaintiff has unreasonably delayed in bringing the action and the delay is prejudicial to the defendant. There is no automatic invocation of laches by a delay of one year before suit is filed. Here, there is no showing that Odivia's delay in filing suit was unreasonable, given that she spent several months trying to find another contractor, nor that Eero has been prejudiced by the delay. Therefore, laches will not provide Eero with a strong defense.

    (C) is incorrect because, if only nominal damages are available, Odivia will not have an adequate legal remedy. Nominal damages are appropriate where there is a breach, but no actual loss. Here, nominal damages would fail to compensate Odivia for the amount she will have to pay above the price agreed to by Eero. Thus, the availability of nominal damages would not, by itself, render specific performance an inappropriate remedy. Regarding

    (D), a court of equity may inquire into the relative values of agreed-upon consideration and deny an equitable remedy if it finds a contract to be unconscionable. Nevertheless, the mere fact that Odivia could not find another contractor to do the job for the price agreed upon by Eero does not establish that her contract with Eero was unconscionable. Thus, Eero will not be successful in his contention that specific performance should be denied on the basis that the price for which he agreed to do the work was too low.

    Rate this question:

  • 24. 

    Jenny, a general contractor, advertised in a trade publication that she planned to bid on the construction of a new building to be located in the Civic Mall. The advertisement welcomed bids from subcontractors to perform various functions, such as plumbing, electrical work, and masonry. The lowest electrical bid was from Ohmco, who bid $20,000. The lowest plumbing bid was from Plunger, who bid $10,000. Jenny used Ohmco's and Plunger's bids in preparing her general bid. At 2 p.m., on June 22, Jenny submitted her general bid. At 3 p.m., Plunger called her and said, 'I'm sorry, Jenny, but I made a mistake on that bid I submitted to you; I can't possibly do that plumbing work for a dime less than $12,000.' Jenny told him, 'I can't do anything about that because I've already submitted my general bid.' Jenny was awarded the contract. Assume for purposes of this question only that after receiving the contract, Jenny hired Flusher to do the plumbing work on the building at a cost of $12,000. She now sues Plunger for damages. Jenny is entitled to recover:

    • A.

      $10,000.

    • B.

      $2,000, which represents the difference between Plunger's bid and the amount Jenny had to pay for plumbing work.

    • C.

      Nothing, because $12,000 was a reasonable amount to pay for the work performed.

    • D.

      Nothing, because Jenny did not accept Plunger's bid before it was withdrawn.

    Correct Answer
    B. $2,000, which represents the difference between Plunger's bid and the amount Jenny had to pay for plumbing work.
    Explanation
    (B) Plunger can be held liable because its offer will be deemed irrevocable for a reasonable length of time on a theory of promissory estoppel. Promissory estoppel renders an offer binding as an option contract even without consideration if the offeror should reasonably expect it to induce action or forbearance of a substantial character by the offeree before acceptance, and such action or forbearance is in fact induced. [Restatement
    (Second) of Contracts |871 Plunger offered to do the work for $10,000 if Jenny was awarded the contract. Plunger should have expected that Jenny would use this figure to prepare her bid and that if she was awarded the contract she would be bound. This is what occurred. The measure of Jenny's damages is the amount she reasonably paid for a substitute performance, less the amount saved as a consequence of the breach; i.e., the $12,000 paid to Flusher minus the $10,000 not paid to Plunger, or $2,000.

    (A) is incorrect because Jenny has not yet paid Plunger anything and has only suffered $2,000 in damages. Jenny, therefore, is only entitled to the measure of damages described above.

    (C) is incorrect because it is irrelevant that Jenny paid a reasonable price for the substitute performance. In the absence of awareness on her part of Plunger's mistake, which is not indicated here, she was entitled to have the work done for the contract price, or to receive damages for the difference between the contract price and the cost of the substitute performance.

    (D) is incorrect because Jenny is entitled to relief even if she did not 'accept' Plunger's bid, because she detrimentally relied on it by using it in her bid.

    Rate this question:

  • 25. 

    Via a circular, WidgeCo, a manufacturer of widgets, sent an offer to Distrucorp, a major wholesaler. WidgeCo offered a standard lot (quantity well-known in the widget trade) of widgets for $8,000. This was a good price, and the president of Distrucorp personally mailed back to WidgeCo Distrucorp's standard printed acceptance form. However, the president wrote in large letters in his own hand on the form, 'Our liability on this contract is limited to $200.' Two days later, the WidgeCo sales manager received the communication from Distrucorp. A week later, WidgeCo had sent no additional communication to Distrucorp. Assuming no additional facts, what is the relationship between the parties?

    • A.

      There is no contract between WidgeCo and Distrucorp, because Distrucorp made a material alteration.

    • B.

      There is a valid, enforceable contract between WidgeCo and Distrucorp, but it is limited to the terms of WidgeCo's offer.

    • C.

      There is a valid, enforceable contract between WidgeCo and Distrucorp, and it contains the additional term because WidgeCo raised no objection.

    • D.

      Distrucorp has sent a valid counteroffer to WidgeCo, which WidgeCo can accept or reject.

    Correct Answer
    B. There is a valid, enforceable contract between WidgeCo and Distrucorp, but it is limited to the terms of WidgeCo's offer.
    Explanation
    (B) An enforceable contract exists between WidgeCo and Distrucorp, but it does not include the limitation of liability added by Distrucorp's president. At common law, any different or additional terms in an acceptance of an offer made the response a rejection and counteroffer. In contracts for the sale of goods, however, U.C.C. section 2-207 substantially alters the common law rule. The proposal of additional terms by the offeree in a definite and timely acceptance does not constitute a rejection and counteroffer. Rather, the acceptance is effective unless it is expressly made conditional on assent to the additional terms. If both parties to the sale of goods contract are merchants, the additional terms will become part of the contract unless
    (i) they materially alter the contract,
    (ii) the offer expressly limits acceptance to the terms of the offer, or (iii) the offeror has already objected to the particular terms, or he objects within a reasonable time after notice of them is received. Here, a sale of goods contract is involved and both parties are merchants. Distrucorp accepted WidgeCo's offer by mailing back a standard printed acceptance form, even though the form contained an additional limitation of liability term added by Distrucorp's president. This limitation of liability provision did not become part of the contract, however, because it would have substantially changed the allocation of economic risks and benefits and impaired an otherwise available remedy. Hence, a valid contract exists between the parties but it is limited to the terms of WidgeCo's offer.

    (A) is incorrect because the material alteration by Distrucorp does not prevent formation of the contract under section 2-207, but the term that produces the material alteration will not be a part of the contract.

    (C) is incorrect. The additional term does not become part of the contract, even though WidgeCo raised no objection to it, because it materially altered the contract. An additional term will only become part of the contract if it is not a material alteration and if the offeror does not object to it within a reasonable time. Hence, WidgeCo's failure to object did not cause the term to be included in the contract.
    (D) is incorrect because under section 2-207, Distrucorp's response to WidgeCo was an acceptance rather than a counteroffer. For the response to have been a counteroffer rather than an acceptance, it would have had to expressly condition acceptance upon WidgeCo's assent to the additional term.

    Rate this question:

  • 26. 

    When Esther, Gray's 21-year-old daughter, finished college, Gray handed her a signed memorandum stating that if she would go to law school for three academic years, he would pay her tuition, room, and board, and would 'give her a $1,000 bonus' for each 'A' she got in law school. Esther's uncle, Miller, who was present on this occasion, read the memorandum and thereupon said to Esther, 'and if he doesn't pay your expenses, I will.' Gray paid her tuition, room, and board for her first year but died just before the end of that year. Subsequently, Esther learned that she had received two 'As' in the second semester. The executor of Gray's estate has refused to pay her anything for the two 'As' and has told her that the estate will no longer pay her tuition, room, and board in law school. In an action by Esther against Miller on account of the executor's repudiation of Gray's promise to pay future tuition, room, and board, which of the following would be Miller's strongest defense?

    • A.

      The parties did not manifestly intend a contract.

    • B.

      Gray's death terminated the agreement.

    • C.

      The agreement was oral.

    • D.

      The agreement was divisible.

    Correct Answer
    C. The agreement was oral.
    Explanation
    (C) Miller's strongest argument is that since the contract, by its own terms, could not be performed within a year, it was subject to the Statute of Frauds and should have been in writing. Since it was not, it is void under the Statute of Frauds.

    (A) would be a possible argument, but it is not as strong as the Statute of Frauds argument. Further, it would probably fail as there does appear to be manifest intent to contract.

    (B) is wrong because if there was a valid contract, it would not have been terminated by Gray's death. Rather, Gray's estate would have been liable for the monies owed.

    (D) is wrong because this was not a divisible contract.

    Rate this question:

  • 27. 

    Upon graduation from high school, Nephew could not decide whether he wanted to go on to college or start working for a living. Uncle told Nephew that if Nephew would attend college, Uncle would pay his tuition and expenses for the full year, and that Uncle would pay him a $1,000 bonus for each 'A' Nephew earned as a final grade in a class. Nephew told Uncle that he would attend college. The next day, Grandfather called Nephew and told him that he had learned of the offer that Uncle had made him and that if Uncle failed to pay Nephew as promised he (Grandfather) would. Nephew attended college and earned 'A's as final grades in three classes. Shortly thereafter, Uncle died, and the executor of Uncle's estate refused to pay Nephew the bonus for each of the three 'A's.Nephew will be unsuccessful in trying to enforce Grandfather's promise because:

    • A.

      ) The contract was illusory.

    • B.

      The contract was oral.

    • C.

      There was no consideration flowing to Grandfather.

    • D.

      The fact that Nephew received a year's worth of free education and tuition was sufficient compensation for Nephew's efforts in earning the three As.

    Correct Answer
    B. The contract was oral.
    Explanation
    (B) Grandfather's promise was a promise to answer for the debt of another, which is required to be in writing under the Statute of Frauds.

    (A) is wrong because the contract clearly is not illusory; Uncle promised to pay if Nephew received 'A's. Uncle would be paying money and Nephew would be giving Uncle the satisfaction of Nephew's receiving 'A's.

    (C) is wrong because the consideration for Grandfather's promise is Nephew's act of getting 'A's.

    (D) is incorrect because this was not the bargained-for exchange Nephew expected.

    Rate this question:

  • 28. 

    Popeye owned a large fleet of buses. He was fortunate enough to have won the contract to bus all the schoolchildren in a large midwestern city and most of its surrounding suburbs. He purchased a number of new vehicles to expand his fleet. Popeye had farmed out the maintenance of the buses to a number of different repair shops, but he now desired the consistent results obtainable from dealing with one repair and maintenance operation. He therefore contracted in writing with Bluto that all general maintenance and extraordinary repairs required for Popeye's buses would be performed by Bluto, who owned and operated 'Bluto's Bus and Truck Center.' A schedule of fees for most standard repairs was included in the contract. It happened that Popeye's wife, Olive, owned and operated a small advertising agency. Popeye wanted Olive's business to succeed, and so he included a clause in his agreement with Bluto that Bluto would place all his ads for his repair shop through Olive's agency during the oneyear term of the agreement. Assume for purposes of this question only that for six months Bluto dutifully placed all his ads through Olive's agency and informed her of his agreement with Popeye. During that time, Olive turned down work from two prospective clients because of the time that she would have to devote to designing and disseminating ads from Bluto. Bluto then discovered that Popeye was having some of his buses repaired and maintained by other shops. Bluto immediately ceased placing ads through Olive and employed another agency. Can Olive successfully bring suit against Bluto to enforce the agreement?

    • A.

      Yes, because she partially performed by placing ads for Bluto during the first six months of the agreement.

    • B.

      Yes, because she detrimentally relied on Bluto's promise to place ads when she refused other clients.

    • C.

      No, because Olive provided no consideration for the agreement between Popeye and Bluto.

    • D.

      No, because Popeye's exclusive use of Bluto's repair shop was a condition for Bluto's duty to purchase ads through Olive, and Popeye's actions excused Bluto's duty to Olive.

    Correct Answer
    D. No, because Popeye's exclusive use of Bluto's repair shop was a condition for Bluto's duty to purchase ads through Olive, and Popeye's actions excused Bluto's duty to Olive.
    Explanation
    (D) Bluto will not be liable to Olive because Bluto's duty to purchase ads was excused by Popeye's failure to use Bluto's repair shop exclusively. The facts indicate that Popeye and Bluto entered into a valid contract that required Bluto to render some performance to Olive. Olive is an intended third-party beneficiary because
    (i) she was expressly designated in the contract,
    (ii) some performance is to be made directly to her, and
    (iii) she stands in such a relationship to the promisee (she is Popeye's wife) that an intent to benefit her can be inferred. She can enforce the contract because her rights have vested; she has materially changed position in justifiable reliance on the promise by turning down work from two prospective clients because of the time Bluto's ads would take. However, when the third-party beneficiary sues the promisor on the contract, the promisor may assert any defenses to formation or performance that he would have been able to assert against the promisee, including failure of a condition. Bluto's promise to perform by placing all of his ads with Olive during the term of the agreement was dependent on the condition that Popeye have all of his buses repaired by Bluto. The failure of Popeye to fulfill this condition excused Bluto's duty to continue to place ads with Olive, and Bluto will be able to assert this defense to performance against Olive.

    (A) is incorrect because Olive's partial performance does not create a duty for Bluto to continue placing the ads. Bluto's duty is also dependent on performance by Popeye.

    (B) is incorrect because Olive's detrimental reliance only establishes that her rights in the contract have vested. Thus, she can bring a contract action against Bluto, but Bluto will have a defense.

    (C) is incorrect because a third party can acquire rights in a contract as an intended beneficiary even though she provided no consideration. The fact that Olive may be a donee beneficiary would not prevent her from enforcing the contract.

    Rate this question:

  • 29. 

    Warden, a wholesaler, sold quality shoes to retailers in a six-state region. Warden ordered 1,000 pairs of shoes from Michael, a shoe manufacturer. The shoes cost $50 per pair, so the total contract price was $50,000. It happened that Michael owed $50,000 to Tyree, a supplier of leather. Michael assigned, in writing, 'all proceeds from the contract with Warden' to Tyree. Michael notified Warden that he had assigned the proceeds of the contract to Tyree and then shipped the 1,000 pairs of shoes to Warden. Upon receipt of the shoes, Warden discovered that 10% of the shoes were defective. He sent a check for 90% of the contract price ($45,000) to Michael. Michael deposited the check. Realizing that the business was in deep trouble, Michael withdrew all funds from his bank account and took off for a country with which the United States has no extradition treaty. Tyree demands payment from Warden. Assume for purposes of this question only that there was no agreement with Tyree, and that Michael neither deposits the check nor departs for a foreign country. Has Warden made a proper rejection?

    • A.

      Yes, because the shoes were defective.

    • B.

      Yes, because he made only partial payment.

    • C.

      No, because Warden accepted the shoes, and failed to seasonably notify Michael of any rejection due to the defects.

    • D.

      No, unless the defects were substantial.

    Correct Answer
    C. No, because Warden accepted the shoes, and failed to seasonably notify Michael of any rejection due to the defects.
    Explanation
    (C) Warden did not properly reject, because he accepted the shoes and failed to give proper notice of rejection. A buyer who receives nonconforming goods generally has the right to accept all, reject all, or accept any commercial units and reject the rest. Here, 10% of the shoes shipped were defective, so Warden had a right to reject. To properly reject, the rejecting party must, within a reasonable time after delivery and before acceptance, seasonably notify the seller of the rejection. If the notice fails to state a defect, the buyer cannot rely on that defect if the seller could have cured by supplying conforming goods. Here, Warden failed to notify Michael of the defects; he merely sent a check for less than the contract price. Since the contract did not have a particular delivery date, Michael probably had time to cure and, presumably, would have. Thus, Warden cannot rely on the defect in claiming a breach; therefore, his rejection was improper.

    (B) is wrong because, as explained above, partial payment did not give notice of the specific defect, and so Warden could not rely on that defect in rejecting. Thus, the rejection was not proper.

    (D) is wrong because the right of rejection exists for any defect, whether substantial or minor. The U.C.C., which governs sales contracts such as this one, gives buyers the right to a perfect tender.

    Rate this question:

  • 30. 

    Owner and Builder executed a contract providing that Builder was to construct a residence on a specified location according to plans and specifications drawn up by Architect. The total contract price was $500,000. The lot on which the residence was to be built was located on the seashore in Palm Beach, Florida and there was an existing woodframe structure that had to be demolished before the residence could be built. Owner contracted with Designer to furnish the interior of the residence after Builder completed construction, but no date was included in either contract for completion of the home. The contract between Owner and Builder stated that construction would begin within two weeks after the existing structure was demolished and the rubble removed from the lot. The contract between Owner and Builder was signed November 12, and the contract between Owner and Designer was signed November 11. Assume for the purposes of this question only that the day after the preexisting structure is demolished and the rubble removed, a severe storm causes gigantic waves that erode the seashore so that Owner's lot is now under water. Must Builder still perform the contract?

    • A.

      No, the contract is void because the subject of the contract was destroyed through no fault of the parties.

    • B.

      No, Builder is discharged of his obligation because of impossibility of performance.

    • C.

      Yes, if Owner obtains an alternative lot within a reasonable period of time.

    • D.

      No, the contract is void because of mutual mistake.

    Correct Answer
    B. No, Builder is discharged of his obligation because of impossibility of performance.
    Explanation
    (B) Prevention of performance by an irresistible, superhuman cause is an excuse for nonperformance of a contract, unless the parties stipulate to the contrary. The destruction of the lot by the forces of nature rendered performance impossible, so Builder need not perform.
    (A) is wrong because destruction of the property does not make the contract void; it merely discharges Builder.
    (C) is a misstatement of law.
    (D) is wrong because there was no mutual mistake.

    Rate this question:

  • 31. 

    Induscorp was a manufacturer of machine tools. Barney, a factory owner, telephoned Induscorp's order department and placed an order for two of Induscorp's standard 'Type-A' machines. Barney and Induscorp came to an oral agreement whereby the total price for both machines was agreed to be $10,000. The first machine was to be delivered on May 1, with payment of $5,000 due 30 days after delivery, and the second machine was to be delivered on June 1 on the same terms (payment of $5,000 due 30 days after delivery). Although Induscorp did not carry the machine in stock, no retooling was required because the Type-A machine was a standard model. The first machine was duly delivered on May 1. The second machine arrived on June 1, but Barney refused to accept delivery and also refused to pay for the first machine. Assume that it cost Induscorp $3,000 to manufacture each Type-A machine, and that Induscorp could resell the machine for only $3,000. If Induscorp sues Barney on June 2, what damages should be awarded aside from any incidental damages?

    • A.

      $3,000.

    • B.

      $5,000.

    • C.

      $7,000.

    • D.

      $10,000.

    Correct Answer
    B. $5,000.
    Explanation
    (B) Induscorp should recover $5,000 because the oral contract between Induscorp and Barney is enforceable to the extent Barney received and accepted the goods. A promise for the sale of goods of $500 or more is not enforceable unless evidenced by a writing signed by the party to be charged. [U.C.C. |2-201(1)] However, an oral contract for such goods is enforceable to the extent of goods received and accepted by the buyer. [U.C.C. |2-201(3)
    (C)] Oral contracts for specially manufactured goods not suitable for sale in the ordinary course of the seller's business also are enforceable when the seller has begun substantially to perform. Here, the parties' agreement was oral. Barney accepted one machine, but neither machine was specially manufactured. Barney is bound to pay the $5,000 contract price for the accepted machine, but he is not bound to pay for the rejected machine.

    (A) is incorrect because Induscorp is entitled to the contract price for the machine Barney accepted, not just restitution. If a contract is unenforceable because of noncompliance with the Statute of Frauds, a party can generally sue for the restitution of any benefit that has been conferred. However, as discussed above, an oral contract for the sale of goods of $500 or more is enforceable to the extent of goods received and accepted by the buyer. Therefore, because Barney accepted one machine, Induscorp is entitled to the $5,000 contract price of that machine, not just $3,000 in restitutionary damages.

    (C) is incorrect because Induscorp is not entitled to any damages as to the rejected machine. As indicated above, Induscorp is entitled to $5,000 for the first machine. There is no enforceable contract regarding the second machine, and Induscorp is not entitled to damages for that machine. If there were an enforceable contract for the second machine,

    (C) would state a proper measure of damages|if a buyer breaches by refusing to accept goods, the seller is entitled to recover the difference between the contract price ($5,000) and the market or resale price
    ($3,000), here, $2,000. Thus, Induscorp would be entitled to $5,000 for the accepted machine and $2,000 for the rejected machine, or $7,000.

    (D) is incorrect for the same reason that

    (C) is incorrect|Induscorp is only entitled to the contract price of the machine accepted by Barney. If the contract for the second machine were enforceable,

    (D) would still not be a proper measure of damages. Under the U.C.C., the seller has a right to force goods on a buyer who has not accepted them only if the seller is unable to resell the goods or if the goods have been lost or damaged after the risk of loss passed to the buyer. [U.C.C. |2-709] Because Induscorp can resell the second machine, which has not been lost or damaged, it could not recover the full price of the second machine from Barney even if the contract was fully enforceable.

    Rate this question:

  • 32. 

    Tarbel, a contractor, entered into a contract with Denhart College to remodel a residence hall during the summer. As specified by the contract, the work had to be completed before the fall semester began at the beginning of September. Because Tarbel received a great deal of other maintenance business from the college, his price of $400,000 was significantly lower than other contractors and he was not going to demand payment until the work was completed. By the end of the first week in August, Tarbel had completed 75% of the project and had expended $350,000 in labor and materials. At that time, however, a labor dispute between Tarbel and his employees prompted most of the workers to walk off the job. Because prospects for a quick settlement of the dispute were doubtful, Tarbel informed the college that he would not be able to meet the completion deadline. A week later, the college obtained another contractor who was able to finish the project by the end of August. The college paid him $150,000, which included a substantial amount of overtime for his workers. The increase in value of the residence hall due to the remodeling was $425,000. Tarbel, who had not been paid, files suit against Denhart College, which files a counterclaim against Tarbel. What should the ultimate recovery be in this action? (

    • A.

      Tarbel should recover nothing from the college because Tarbel breached the contract.

    • B.

      Tarbel should recover $200,000 in quasicontract from the college, which is the difference between its expenditures and the amount the college paid the other contractor to complete the work.

    • C.

      Tarbel should recover $250,000 in quasicontract from the college, which is the contract price minus the amount the college paid the other contractor to complete the work.

    • D.

      Tarbel should recover $275,000 in quasicontract from the college, which is the difference between the value of the completed remodeling and the amount the college paid the other contractor to complete the work.

    Correct Answer
    C. Tarbel should recover $250,000 in quasicontract from the college, which is the contract price minus the amount the college paid the other contractor to complete the work.
    Explanation
    (C) Tarbel should be able to recover $250,000 in quasi-contract. Where a builder in a construction contract breaches during the construction, the owner of the building is entitled to the cost of completion plus compensation for any damages caused by the delay in completing the building. Most courts, however, at least where the breach is not willful, will allow the builder to offset or recover for work performed to date to avoid the unjust enrichment of the owner. This recovery, which would be based on quasi-contract, is usually based on the benefit received by the unjustly enriched party. If substitute performance is readily obtainable, damages are measured by the unpaid contract price minus the cost of completion
    (up to the value of the benefit received by the defendant). Here, Tarbel's duty to complete the project was not discharged by impossibility; he could have hired another contractor to take his place or yielded to his employees' demands. Hence, Tarbel's failure to complete the remodeling constituted a breach of contract and resulted in the college having to expend $150,000 to have the building completed in time. However, Tarbel did not receive any payments for the work that he did before breaching; the college would be unjustly enriched if it does not have to pay for any of this work. The benefit of the completed remodeling is measured by the contract price, $400,000, because a quasi-contract recovery here would be based on the failed contract between the parties and substitute performance is readily obtainable. This amount is reduced by the $150,000 cost of completion that the college can recover from Tarbel, leaving a net recovery of $250,000 for Tarbel.

    (A) is incorrect. Because Tarbel's breach was not willful, most courts would permit him to recover in quasi-contract to prevent the college's unjust enrichment from the work that he did.

    (B) is incorrect because recovery measured by the claimant's detriment (i.e., his reliance interest) is only an appropriate alternative where the standard 'benefit' measure would achieve an unfair result; it is not applied where the party seeking quasi-contract recovery was the breaching party.

    (D) is incorrect because courts will always limit relief to the contract price where the claimant is the breaching party. Measuring the benefit to the college in terms of the value of the improvements rather than the contract price will deny to the college the benefit of the bargain that it became entitled to when Tarbel breached.

    Rate this question:

  • 33. 

    Yott was a wealthy sportsman and sailing enthusiast. He purchased a large old wooden sailing ship for $200,000. Although the boat was a classic, Yott wanted the vessel to be modernized and made more comfortable. Yott entered into a written contract with Salty, a marine architect and engineer. Under the terms of the contract, Yott agreed to pay Salty $7,500, and Salty agreed to draw up the modernization plans and to be the contractor for the modernization of the ship. Assume for purposes of this question only that at the time Yott and Salty entered into the agreement, Yott told Salty orally, 'Of course, your modernization plan is subject to the approval of my sister, Marina. Although I'm sure she'll like any design of yours, we've got no deal unless your plans meet with her approval.' Salty agreed to this. Salty finished his drawings and submitted them to Yott, who was enthusiastic about Salty's designs. Marina, a famous yachtswoman, was engaged in a trans-Pacific yacht race at the time and was not expected home for a number of weeks. Cheered by Yott's enthusiasm, Salty went ahead and modernized the ship according to his designs. When Salty finished the work, he submitted a bill to Yott. Yott refused to pay, pointing out that Marina had never approved the designs. If Salty sues Yott, which of the following issues of contract law is most likely to be decisive in determining the outcome of the case?

    • A.

      Statute of Frauds.

    • B.

      Parol evidence rule.

    • C.

      Rules of construction.

    • D.

      Conditions precedent.

    Correct Answer
    D. Conditions precedent.
    Explanation
    (D) Approval of the modernization plans by Marina is a condition precedent because without such approval the parties have no agreement. Where there is an oral condition precedent, evidence of the condition falls outside the parol evidence rule. The parol evidence rule provides that where the parties to a contract express their agreement in a writing with the intent that it embody the full and final expression of their bargain, any other expressions, written or oral, made prior to the writing, as well as any oral expressions contemporaneous with the writing, are inadmissible to vary the terms of the writing. Certain forms of extrinsic evidence are deemed to fall outside the scope of the parol evidence rule. For instance, a party to a written contract can attack the validity of the agreement. One way of doing so is by asserting that there was an oral agreement that the written contract would not become effective until the occurrence of a condition. Such a condition would be deemed a condition precedent to the effectiveness of the agreement, and evidence of the condition will be freely offered and received. Here, Yott and Salty have entered into a written agreement that apparently embodies the full and final expression of their bargain. However, Yott's oral statement at the time of entering into the agreement indicates quite clearly that the parties had no agreement absent the approval of Marina, and Salty agreed with this statement. Thus, there is an oral agreement that the written contract would not become effective until the occurrence of a condition precedent. As discussed above, evidence of this oral condition does not come within the purview of the parol evidence rule and is therefore admissible. Consequently, Yott can assert the nonoccurrence of a condition precedent as a way to avoid liability on the contract.

    (B) is incorrect because, as explained above, the nature of the oral agreement takes it outside the scope of the parol evidence rule. Therefore, the rule will not be decisive in determining the outcome of this case.

    (A) is incorrect because the agreement at issue here is not of a type that is covered by the Statute of Frauds. The agreement does not involve:
    (i) a promise by an executor or administrator to pay the estate's debts out of her own funds;
    (ii) a promise to answer for the debt of another;
    (iii) a promise made in consideration of marriage;
    (iv) a promise creating . an interest in land;
    (v) a promise that by its terms cannot be performed within one year; or
    (vi) a promise for the sale of goods at a price of $500 or more. Therefore, the Statute of Frauds is inapplicable to these facts.

    (C) is incorrect because the term 'rules of construction' refers to rules used by courts when interpreting contracts; e.g., contracts are to be construed as a whole, words are to be construed according to their ordinary meaning, or custom and usage in the particular business and locale should be considered. The facts and issues presented by this question do not call for the application of any such principles of contract interpretation.

    Rate this question:

  • 34. 

    After his widowed father's death, Son undertook the task of cleaning out Father's condominium and preparing it for resale. After removing valuables and sentimental objects and packing up clothes and other household items that were in good enough shape to donate to charity, Son called Ye Olde Purveyor of Junk ('Purveyor') to remove everything else. Son entered into a written agreement with the company to completely empty out the condo and wash the walls, floors, and appliances. The parties agreed on a contract price of $1,500. Shortly after beginning performance, Purveyor assigned to Munson its right to all monies due under the contract (i.e., $1,500), and Munson promptly notified Son of the assignment and Son acknowledged the same. Purveyor continued working, completely emptying out the condo and washing the walls, floors, and all of the appliances except for the oven before quitting the job. It would cost $150 to hire a substitute to clean the oven. If Munson sues Son, Munson would be entitled to recover: (A)(B) (C)(D)

    • A.

      $1,500, the amount assigned, and Son may look to Purveyor to recover for the minor breach.

    • B.

      The reasonable value of the labor and materials expended by Purveyor on the portion of the job it did complete.

    • C.

      $1,350, on a theory of substantial performance.

    • D.

      Nothing, because Son's duty to pay is subject to a constructive condition precedent, and the assignee takes subject to the defense that the condition has not been satisfied.

    Correct Answer
    C. $1,350, on a theory of substantial performance.
    Explanation
    (C) Munson will be able to recover the contract price less damages for Purveyor's minor breach. Generally, an assignee has whatever rights his assignor would have against the obligor. Similarly, the assignee is subject to any contract-related defenses that the obligor has against the assignor. Thus, Munson will have whatever rights Purveyor would have against Son. Here, Purveyor completed all of the tasks that needed to be done except for one. If the work remaining on the contract is minor, Purveyor will be seen as substantially performing its contract, and substantial performance will discharge its duty to perform and obligate payment by Son. Since the facts state that the cost of finishing the job was relatively small
    (10% of the cost of the contract), it will probably be seen as a minor breach. Thus, Son cannot avoid payment of the contract price. However, despite the substantial performance, the other party to the contract may recover damages for the less than complete performance. Thus, Son will be able to offset his damages from the breach. Munson then will be able to recover $1,350
    (the contract price less the damages).

    (A) is incorrect because, as stated above, the obligor may offset damages directly against the assignee; he does not have to pay the full contract price and then seek damages from the assignor.
    (B) is incorrect because this suggests a quasi-contract remedy, but as stated above, Munson, the assignee of Purveyor, would be able to recover the contract price less damages because Purveyor substantially performed.

    (D) is incorrect because the constructive condition precedent to Son's duty to pay
    (Purveyor's performance) has been satisfied here by substantial performance.

    Rate this question:

  • 35. 

    Shortly after a series of rapes took place within Big City, the city council of Big City approved the offering of a $25,000 reward for the arrest and conviction of anyone guilty of the rapes. Information concerning the reward was broadcast by Big City's sole radio station once daily for one week. Shortly thereafter, Karen, a victim of one of the rape attacks, hired Stevens, a private detective, to find the person responsible for perpetrating the rapes. Karen had agreed to pay Stevens $100 per each day spent investigating the rapes. The next year, the city council of Big City passed a resolution repealing its reward offer. The city council caused this resolution to be telecast once daily for a week over Big City's local television station, the local radio station, meanwhile, having ceased operations. One month later, Philip voluntarily confessed to Stevens to having committed all of the earlier rapes. Philip was arrested and ultimately convicted of all the rapes. Karen thereupon paid Stevens at the agreed-on rate and suggested also that Stevens claim the city's $25,000 reward, of which Stevens had been previously unaware. Stevens immediately made the claim. The city refused to pay Stevens anything, although he swears that he never heard of the city's repeal of the reward offer before claiming the reward. Assume that Big City has no immunity to suit. In which of the following ways could Big City's reward offer be effectively accepted?

    • A.

      Only by an offeree's making the arrest and assisting in the successful conviction of a rapist within the scope of the offer.

    • B.

      Only by an offeree's return promise to make a reasonable effort to bring about the arrest and conviction of a rapist within the scope of the offer.

    • C.

      By an offeree's communication of assent through the same medium (radio) used by the city in making its offer.

    • D.

      By an offeree's supplying information leading to arrest and conviction of a rapist within the scope of the offer.

    Correct Answer
    D. By an offeree's supplying information leading to arrest and conviction of a rapist within the scope of the offer.
    Explanation
    (D) The language of the reward offer should not be read with total literalness, but rather in the context of what Big City was seeking to obtain and the normal duties required to accept an offer of this kind. Therefore,

    (A), which would require that the offeree actually participate in the arrest, can be eliminated. Offers of this type normally require only that the offerees supply information leading to the arrest and conviction of the culprit.

    (B) and

    (C) can be ruled out without difficulty because they would envision a bilateral contract under which the offeree would be promising or assenting to Big City's proposal. In this kind of word context the offer is obviously the unilateral proposal that can be accepted only by performance.

    Rate this question:

  • 36. 

    Winston went to Rudder, a boatbuilder, and told Rudder that he wanted a yacht built to his specifications. They agreed that the price would be $400,000, and that Winston was to make payment in full within 30 days after Winston had accepted delivery of the yacht. They further agreed that Rudder would not subcontract any of the work. Rudder, however, contacted Genoa, a master sailmaker, and subcontracted the sails for the yacht to Genoa. They agreed orally that Rudder would pay Genoa $25,000 for the sails within 20 days of receiving them. Rudder did not tell Genoa of his agreement with Winston regarding subcontracting. Genoa made the sails and delivered them to Rudder, who then completed the yacht and delivered the boat to Winston. Although the yacht was built to his specifications, he refused to accept it after he learned that Rudder had subcontracted for the sails. When the 20-day payment period for the sails had expired, Genoa went to Rudder and demanded the $25,000. Rudder told Genoa that he could not pay the $25,000 unless Winston paid for the yacht. If Genoa decides to sue, is there an enforceable contract between Rudder and Genoa?

    • A.

      Yes, because they are merchants under the Uniform Commercial Code.

    • B.

      Yes, because Genoa fully performed.

    • C.

      No, because Rudder had agreed not to subcontract. May Jim cancel the contract?

    • D.

      No, because of the Statute of Frauds.

    Correct Answer
    B. Yes, because Genoa fully performed.
    Explanation
    (B) There is an enforceable contract between Genoa and Rudder because Genoa fully performed. The contract here was for the sale of goods
    (sails) for the price of $500 or more; thus, the contract is within the Statute of Frauds. A contract within the Statute of Frauds is generally unenforceable absent a memorandum signed by the party to be charged containing the contract's essential terms. However, there is an exception to the general rule for goods received and accepted. Here, although the contract was oral, Rudder accepted the sails, and so he is bound despite the Statute. Note that Rudder might also be bound under another exception to the Statute|for specially manufactured goods if the sails were made specially for the yacht and were not suitable for sale to others.

    (A) is incorrect because the U.C.C. does not exempt merchants from the Statute of Frauds. Although it provides a special confirmatory memo rule by which a merchant may be bound even if his signature does not appear on the writing evidencing the contract, here there is no writing at all.

    (C) is incorrect because it is irrelevant. The fact that Rudder agreed not to subcontract is relevant to whether he breached his contract with Winston, but it does not affect his contract with Genoa.

    (D) is incorrect because, as stated above, the contract here falls within an exception to the Statute of Frauds because Rudder accepted the sails.

    Rate this question:

  • 37. 

    Indegas wanted to ensure that it had sufficient stocks of gasoline and oil for its independent dealerships; it therefore entered into contracts with various refineries to purchase the gasoline they produced. One such agreement was with Cal-Tex, whereby Indegas was given the right to purchase all gasoline refined by Cal-Tex for the next five years at a price set at 95% of the domestic market price at the time of delivery. Indegas agreed to purchase no less than 5,000 gallons a week and to use its own tankers to transport the gasoline from Cal-Tex's refinery to its storage facilities. At the time this contract was signed, Indegas gave written notice to Cal- Tex that it intended to buy all gasoline produced by Cal-Tex until further notice. For the first year, Indegas continued to purchase all gasoline produced by Cal-Tex. However, at the end of that year Cal-Tex purchased an out-of-state refinery that refined all its gasoline from domestic shale oil. As a result of this merger and of more efficient plant methods, Cal- Tex doubled its production of gasoline by the next year. At a meeting between Cal-Tex and Indegas, Cal-Tex's president noted that Indegas was getting as much gasoline as it needed, and that Cal-Tex intended to sell the extra gasoline it was producing on foreign markets at a higher price than Indegas was paying. Indegas agreed with this proposal and signed an addendum to the original agreement reflecting this change. Thereafter, Indegas continued to purchase one-half Cal-Tex's total capacity until the following year when, as a result of a foreign oil boycott, Cal-Tex was unable to purchase sufficient foreign oil to operate profitably its local refinery. Cal-Tex, by letter, notified Indegas that it could no longer deliver gasoline to it in accordance with their agreement. Nevertheless, Cal-Tex's other refinery was able to increase its production by 25% and continue to sell to foreign markets. Assume that the oil boycott ended, but that Cal-Tex sold both its refineries to a large oil company. What is the effect of this sale on Cal-Tex's obligation to Indegas?

    • A.

      The sale discharges its obligation to Indegas because there has been a full performance

    • B.

      Cal-Tex is liable for damages if the other oil company fails to deliver gasoline to Indegas

    • C.

      Cal-Tex is excused from further performance because it has no more refineries to produce gasoline.

    • D.

      Cal-Tex breached its contract with Indegas.

    Correct Answer
    B. Cal-Tex is liable for damages if the other oil company fails to deliver gasoline to Indegas
    Explanation
    (B) Since delivery of gas is not personal in nature, that duty can be assigned. The quantity will be measured by Cal-Tex's original output. However, when a duty is delegated to a delegate, the delegator remains liable should the delegate fail to perform.

    (A) is incorrect because the contract was for five years, and five years have not yet elapsed.

    (C) would be correct only if Cal-Tex went out of business without delegating its duties to another by selling the other the business, not the case here.

    (D) is wrong because, as indicated above, such a delegation is proper.

    Rate this question:

  • 38. 

    On July 1, Sam offered to sell Durham, his prize bull, to Bill for $15,000. In return for $100 which Bill paid Sam, Sam handed to Bill a signed written statement that recited the offer and stated that Sam promised not to revoke for a period of 30 days. Both Bill and Sam are cattle ranchers. On July 10, Bill wrote Sam as follows: 'I cannot pay more than $12,500 for Durham. If you cannot accept this amount, I will not be able to go through with the deal.' Bill did not know that Sam had already sold Durham to another party on July 9. On July 27, Bill wrote Sam: 'I accept your offer to sell Durham. Enclosed is my check for $15,000.' Sam received Bill's letter of July 27 on July 29. Assume Bill's letter of July 10 had read: I have decided to take Durham. A check for $15,000 is enclosed. I am leaving for Canada for six months and will pick up Durham on January 1. I will pay you for his board and care. Bill's letter is: (A) (B)(C) (D)

    • A.

      A counteroffer, because it changes the terms of the offer.

    • B.

      A counteroffer, because it was not a definite expression of acceptance.

    • C.

      An acceptance, and Sam must board the bull but is entitled to the reasonable value of that service.

    • D.

      An acceptance, and Sam may refuse to board the bull.

    Correct Answer
    D. An acceptance, and Sam may refuse to board the bull.
    Explanation
    (D) Under U.C.C. section 2-207, proposed new terms in a deal between merchants do not terminate the offer
    (i.e., do not constitute a counteroffer) and they become part of the deal unless
    (i) the offer is limited to its own terms,
    (ii) the new terms materially alter the deal, or
    (iii) the offeror objects within a reasonable time.

    (D) is correct and

    (C) is incorrect because Bill's proposal would materially alter the terms of the contract.

    (A) and

    (B) are wrong because the letter is not a counteroffer.

    Rate this question:

  • 39. 

    Cordero owned a beautiful home entertainment center, which included a large-screen television. Cordero worked very hard at two jobs, so he had little opportunity to enjoy the entertainment system. Cordero's friend, Tina, wanted an entertainment system so that she could watch her favorite daytime soap operas. She importuned Cordero, and he finally agreed to sell her his home entertainment system for $2,500. Tina made a downpayment of $700 and took possession of the entertainment system. She agreed to pay the balance due to Cordero in 18 equal $100 installments, beginning on June 5, with subsequent installments due on the fifth of each month until the balance was paid in full. Gabrielle owed Tina $2,000. On May 20, Tina entered into an oral agreement with Gabrielle whereby Gabrielle agreed to make the 18 $100 installment payments to Cordero. In exchange for this, Tina promised to forgive Gabrielle's $2,000 debt to her. On June 7, Cordero called Tina and asked where his first $100 installment payment was. At that time Tina told Cordero of her agreement with Gabrielle. Gabrielle has made none of the installment payments. Does Cordero have a right to recover the debt from Tina?

    • A.

      Yes, if Cordero has not yet filed suit against Gabrielle.

    • B.

      No, unless Cordero is not able to recover the money from Gabrielle.

    • C.

      Yes, unless Cordero has expressly agreed with Gabrielle and Tina that Gabrielle would assume Tina's duties under the contract.

    • D.

      No, because Gabrielle expressly assumed Tina's duties under the contract

    Correct Answer
    C. Yes, unless Cordero has expressly agreed with Gabrielle and Tina that Gabrielle would assume Tina's duties under the contract.
    Explanation
    (C) Tina will remain liable to Cordero for the debt unless the parties have agreed to a novation. When a party to a contract delegates her duties to another, the original party
    (the delegator) remains liable on her contract. This is so even if the delegate expressly assumes the delegator's duties. However, a different result would obtain if the obligee the person to whom the duties are owed) expressly consents to this transfer of duties. This would constitute an offer of novation. A novation occurs when a new contract substitutes a new party to receive benefits and assume duties that had belonged to one of the original parties under the terms of the old contract. For a novation, all parties must agree that the contractual duties between the original contracting parties are extinguished; a valid and enforceable new contract takes the place of the previous contract. The effect of the novation is to discharge the old contract. Here, Gabrielle has expressly assumed Tina's duty to pay Cordero, but that does not discharge Tina's liability to Cordero. Cordero may still sue Tina on the existing obligation between them. However, if the parties expressly agree to the substitution of Gabrielle for Tina, a novation would occur and the original contract would be extinguished. In that case, Tina would be discharged of all of her duties to Cordero regardless of whether Gabrielle subsequently performs her duties.

    (A) is incorrect because Cordero, who is a third-party beneficiary of the assumption agreement between Tina and Gabrielle, does not need to elect between the third-party promisor(Gabrielle) and his own debtor (Tina). He may sue both of them (although he can recover only one satisfaction). Thus, whether Cordero has filed suit against Gabrielle is irrelevant to his right to recover the money from Tina.
    (B) is incorrect for the same reason. Tina cannot change her status to that of a guarantor simply by delegating her duties to Gabrielle. Cordero can choose to sue Tina on the existing obligation between them without first trying to recover from Gabrielle.

    (D) is incorrect because, as discussed above, the fact that the delegate(Gabrielle) expressly assumed the duties of the delegator (Tina) does not discharge the delegator's liability to the obligee Cordero).

    Rate this question:

  • 40. 

    Mary's doctor informed her that she had a rare blood disease that was almost always fatal. He further informed her that there was no treatment known to medical science for this disease. Out of desperation, Mary consulted Quack, who claimed to have a cure for the blood disease. Mary entered into an agreement with Quack under which Quack promised to treat Mary for the blood disease. However, no price was given for the treatment. After two months of treatment, Mary did not appear to have improved at all. Mary's father, Stu, went to see Quack and told Quack that if Quack would cure Mary of the blood disease, Stu would pay Quack $25,000. Four months later, after weekly sessions with Quack, Mary again went to see her doctor. This time her doctor told her that she appeared to have recovered completely from the blood disease, because all tests for the disease proved negative; and that, in his opinion, she was completely cured. Assume for the purposes of this question only that Mary refused to pay Quack anything. Quack brings suit against Mary for services rendered. Quack will:

    • A.

      Recover whatever amount Quack shows is his normal fee for the treatment.

    • B.

      Recover a reasonable price for his services.

    • C.

      Not recover, because no price term was contained in the original contract.

    • D.

      Not recover, because Quack cannot prove that he was the cause of Mary's recovery.

    Correct Answer
    B. Recover a reasonable price for his services.
    Explanation
    (B) Quack will be able to recover a reasonable price for his services. While the parties failed to agree on a material term, most courts today will imply reasonable terms if they are consistent with the parties' intent as otherwise expressed. Terms that can be supplied by a reasonableness standard include a price term for the performance of services. Unless the parties have shown at the time of contracting that they do not want a contract until they agree on a price, a reasonable price will be implied.
    (A) is wrong because Quack's fee is not necessarily reasonable although the court will take into account Quack's normal fee when it determines the reasonable fee.
    (C) is wrong because it fails to take into account that a court will imply a reasonable price term.
    (D) is wrong because it is irrelevant. In contract, a party need not prove that the 'performing' party actually caused the performance. Mary received the performance she bargained for here|a cure.

    Rate this question:

  • 41. 

    Claw died, leaving his nephew Scratch as his sole heir. Among the items inherited by Scratch were some old oil paintings that had been stored in Claw's attic for a number of years. Scratch knew nothing about art and had no place to put them in his home. He placed an ad in the paper offering to sell the paintings 'at a price to be mutually agreed upon.' Fang, a buyer for an art gallery, responded to the ad and examined the paintings. From the signature and the style, Fang recognized that the artist was William Hamilton, a renowned 19th century American portrait artist. Scratch and Fang agreed upon a price and executed a contract. Which of the following facts, if true, would give Fang the best basis for rescinding the contract with Scratch? (A) (B) (C)(D)

    • A.

      Several of the paintings cracked when they were being transported by Fang because they were brittle with age.

    • B.

      The day after the purchase, a respected art historian announced in a press release that many William Hamilton paintings were actually done by his students and other associates, causing the value of all William Hamilton paintings to decline.

    • C.

      Because of some experimental pigments that the artist had used, the colors began to fade rapidly as soon as the paintings were exposed to light; within a few days, virtually all of the colors had faded away.

    • D.

      The gallery for which Fang had procured the paintings was destroyed by a fire shortly after the contract was executed.

    Correct Answer
    C. Because of some experimental pigments that the artist had used, the colors began to fade rapidly as soon as the paintings were exposed to light; within a few days, virtually all of the colors had faded away.
    Explanation
    (C) The circumstances of (C)offer the best grounds for rescinding the contract based on mutual mistake. When both parties entering into a contract are mistaken about facts relating to the agreement, the contract may be voidable by the adversely affected party if
    (i) the mistake concerns a basic assumption on which the contract is made;
    (ii) the mistake has a material effect on the agreed-upon exchange; and
    (iii) the party seeking avoidance did not assume the risk of the mistake. Here, both parties believed that the paintings would be suitable for viewing and had no reason to suspect that their color would fade away when exposed to light. This occurrence probably rendered the paintings virtually worthless, creating a material imbalance in the exchange. Finally, despite Fang's superior knowledge of the subject matter, it is doubtful that he would be deemed to have assumed the risk of what occurred. In contrast to this situation, the circumstances in
    (A) are not as strong a basis for avoidance. Even assuming that both parties mistakenly believed that the paintings were not too fragile to be transported, that risk is more likely to be deemed assumed by Fang. The circumstances in

    (B) suggest only that Fang may have made a mistake as to the value of the paintings, but since Scratch knew nothing about the identity of the artist, the mistake is unilateral and Fang cannot rescind on this ground. Choice

    (D) is incorrect because the circumstances do not satisfy the requirements for discharge by frustration. Frustration will exist where the purpose of the contract has become valueless by virtue of some supervening event not the fault of the party seeking discharge. To establish frustration, the following elements must be shown:
    (i) there is some supervening act or event leading to the frustration;
    (ii) at the time of entering into the contract, the parties did not reasonably foresee the act or event occurring;
    (iii) the purpose of the contract has been completely or almost completely destroyed by this act or event; and
    (iv) the purpose of the contract was realized by both parties at the time of making the contract. Here, it is doubtful that the purpose for which Fang made the contract was destroyed by the destruction of that one gallery, and there is no indication at all that Scratch was aware of Fang's purpose in making the contract. Thus, Fang cannot raise frustration of purpose as a defense to the contract.

    Rate this question:

  • 42. 

    Mary had a small shop where she sold her own jewelry creations. She was commissioned by a lovestruck young man, Willie Darcy, to design and create a set of rings(engagement and wedding) for Willie's beloved, Elizabeth. Mary designed and created the rings in 18k gold, leaving room in the engagement ring for a large marquise-shaped diamond. She then entered into an oral agreement with Hanson, a well-known gemologist, to provide the diamond, for which Mary would pay Hanson $20,000. Mary's agreement with Hanson was that she would pay him when Willie paid her. Hanson took much care in finding a suitable gemstone, cutting the diamond to show off its brilliance, and otherwise preparing it per Mary's specifications. Hanson delivered the diamond to Mary, who accepted it with praise. Hanson waited to be paid, and when he was not, he contacted Mary, but she refused to pay him, arguing that their agreement was unenforceable and, anyway, Willie has not paid her. If Hanson sues Mary, how much can he recover?

    • A.

      The fair market value of his work, under a quasi-contract theory.

    • B.

      The cost of materials and labor, under a quasi-contract theory.

    • C.

      $20,000, the contract price.

    • D.

      Nothing, because Willie has not paid Mary.

    Correct Answer
    C. $20,000, the contract price.
    Explanation
    (C) Hanson will be able to recover the full $20,000 contract price. Under the U.C.C., the contract is enforceable, despite the absence of a writing, to the extent of the goods accepted, which here is the entire amount contracted for. The proper remedy is the agreed-upon price of $20,000, which Hanson will be able to prove by parol evidence. [See U.C.C. |2-201, comment 2]

    (A) is incorrect because the recovery will be under the contract, because the promises are enforceable under two exceptions to the Statute of Frauds
    (i.e., Mary accepted the diamond, and the diamond was specially shaped and cut for Mary and may not be suitable for sale to others); thus, the quasicontract remedy need not be applied. Note that if the contract had been unenforceable, quasicontract would be a basis for a recovery.

    (B) is incorrect because it also is a possible quasi-contract measure of damages, and as stated above, Hanson will be able to recover under the contract here.
    (D) is incorrect because the court will construe Mary's agreement as a promise rather than an implied condition. If it were a condition, Mary would not have a duty to pay because she was not paid. However, where an agreement provides that a duty is to be performed once an event occurs, if the event is not within the control of the promisee, it is less likely that he will have assumed the risk of its nonoccurrence and therefore less likely to be a condition of the promisor's duty to perform. In doubtful situations, courts will more likely hold that the provision is a promise rather than a condition because this supports the contract and preserves the reasonable expectations of the parties.

    Rate this question:

  • 43. 

    Johnston purchased 100 bolts of standard blue wool, No. 1 quality, from McHugh. The sales contract provided that Johnston would make payment prior to inspection. The 100 bolts were shipped and Johnston paid McHugh. Upon inspection, however, Johnston discovered that the wool was No. 2 quality. Johnston thereupon tendered back the wool to McHugh and demanded return of his payment. McHugh refused on the ground that there is no difference between No. 1 quality wool and No. 2 quality wool. What is Johnston's remedy because the wool was nonconforming?

    • A.

      Specific performance.

    • B.

      Damages measured by the difference between the value of the goods delivered and the value of conforming goods.

    • C.

      Damages measured by the price paid plus the difference between the contract price and the cost of buying substitute goods.

    • D.

      None, since he waived his remedies by agreeing to pay before inspection.

    Correct Answer
    C. Damages measured by the price paid plus the difference between the contract price and the cost of buying substitute goods.
    Explanation
    (C) Where the seller does not deliver or the buyer properly rejects or revokes acceptance of tendered goods, the buyer's basic remedy is the difference between the contract price and either the market price or the cost of buying replacement goods
    (cover).

    (C) reflects the latter measure of damages.
    (A) is incorrect because specific performance is only available where the goods are unique or in other proper circumstances. Here, the wool does not appear to be unique.

    (B) is incorrect because it sets forth a measure of damages that is properly applicable where the buyer accepts goods that breach one of the seller's warranties; here, the goods have not been accepted.

    (D) is incorrect because, as explained in the answer to the preceding question, agreeing to pay before inspection does not waive any of the buyer's rights.

    Rate this question:

  • 44. 

    On March 5, Triton Machine Works mailed the following signed communication to Banfield Machine Tools: 'We hereby offer to sell you 500 diamond core drill bits, priced at $300 each. This is a firm offer.' This offer was received by Banfield Machine Tools on March 7. On March 8, Triton mailed a notice to Banfield stating: Please be advised that our offer of March 5 is revoked. This notice of revocation was received by Banfield on March 10. On March 11, Banfield wired Triton: 'Consider you bound by your offer. Please enter our order as per your offer for 10 diamond core drill bits. Box two per case, hardwood casing only. Our draft for $3,000 has been sent under separate cover.' If Triton defends on the ground that its offer was for the sale of 500 diamond core drill bits in one lot only, rather than in small quantities (here, only 10 bits ordered by Banfield), the court should hold: (A) (B) (C) (D) .

    • A.

      Since Triton's offer failed to state that it was a single lot only, it was subject to partial acceptance at the buyer's option.

    • B.

      Triton should prevail because the offer specifically offered 500 drill bits and it would be commercially unreasonable to hold the seller to piecemeal sales at the stated price.

    • C.

      The attempted formation of a contract is flawed by ambiguity so that the minds of the parties never met and no enforceable obligation to sell was created by Banfield's acceptance.

    • D.

      Parol evidence is admissible on the question of whether the offer contemplated sale of the bits as a single lot or piecemeal

    Correct Answer
    D. Parol evidence is admissible on the question of whether the offer contemplated sale of the bits as a single lot or piecemeal
    Explanation
    (D) The court should admit parol evidence to construe the ambiguous terms of the contract. The terms of a contract that are set forth in a writing intended as a final expression of the parties' agreement cannot be contradicted by evidence of any prior agreement or contemporaneous oral agreement. However, the terms of a contract may be explained or supplemented by parol evidence. [U.C.C. |2-202] The offer does not indicate whether the drill bits are to be sold only as a single lot. Therefore, to clear up the ambiguity in Triton's offer, the court could consider trade practices, past course of dealing, etc., in determining whether Banfield had a reasonable expectation of being able to purchase the goods piecemeal.

    (A) is incorrect because such a narrow interpretation is not clear from the ambiguous language in Triton's offer. When a term of a contract is ambiguous in a writing, the term may be explained by course of dealing, usage of trade, or course of performance. Because it is unclear from the language whether Triton's offer contemplated sale of the bits as a single lot or piecemeal, the offer is not subject to partial acceptance at the buyer's option, and the court may consider parol evidence to clear up the ambiguity.
    (B) is incorrect because it is not clear from the language of Triton's offer whether the bits were being offered as a single lot only. As discussed above, because the language of Triton's offer is ambiguous as to whether sale of the bits as a single lot or piecemeal was contemplated, Triton is not entitled to judgment solely on the basis of the written offer. Instead, the court will admit parol evidence, such as trade practices and past course of dealing, to clear up the ambiguity.

    (C) is incorrect because the court may consider other evidence to clear up ambiguities. As discussed above, the U.C.C. provides that a written contract's terms may be explained or supplemented by (i) course of dealing or usage in the trade, or
    (ii) the course of performance to date, even if the terms appear to be unambiguous. Therefore, Triton's ambiguous offer to sell the bits, which comes under the U.C.C., is not fatal to the formation of the contract because parol evidence is admissible on the question of whether the offer contemplated a piecemeal sale of the bits.

    Rate this question:

  • 45. 

    Martha needed a new pair of shoes. She went to her local Skysheim shop and told the salesperson that she worked in the city's downtown area and had to walk eight blocks to get from her house to her commuter train and then six blocks from her train to her office. During the workday, she had to climb up and down stairs several times. She wanted shoes that were suitable for walking on concrete, had gripping power for stairs, and were comfortable. The salesperson went into his stockroom and brought out four different styles of Skysheim's 'Clouds,' reputably the most comfortable shoe on the market, designed for the type of use that Martha had in mind. Martha tried on each of the four pairs but did not like the way any of them looked. While walking around the store, however, Martha saw a shoe she did like --- 'Hobblers',' Skysheim's high-fashion shoe. She told the salesperson to bring her a pair to try. He did so and explained to Martha that Hobblers were completely made of the finest leather and would probably last for years. Martha tried on the shoes and told the salesperson that she would take them. Assume for this question only that Martha paid for the shoes and walked out of the store. A few moments later, she sat down to look at her new shoes in the sunlight. On close inspection, she noticed a small nick in the leather of one of the shoes. She immediately went back to Skysheim and demanded a refund. The salesperson refused. If Martha sues for a refund, who will prevail?

    • A.

      Martha, because there was a breach of contract.

    • B.

      Martha, because she had a reasonable time after purchase in which to inspect.

    • C.

      Skysheim, because Martha accepted the goods.

    • D.

      Skysheim, because Martha did not give written notice of the breach.

    Correct Answer
    C. Skysheim, because Martha accepted the goods.
    Explanation
    (C) Once a buyer has accepted goods, her right to reject for nonconformance generally lapses and her only remedy is a suit for damages. Acceptance usually occurs when the buyer takes possession of the goods. In some cases, the buyer can revoke acceptance, but the breach must be substantial and the buyer must have a good reason for accepting the goods
    (i.e., something more than not taking the time to carefully inspect). Here, Martha accepted the goods and the breach appears minor.

    (A) entitles Martha only to damages, not a full refund.

    (B) is a misstatement of the law.
    (D) is wrong because written notice is not required; oral notice is acceptable.

    Rate this question:

  • 46. 

    On March 5, Triton Machine Works mailed the following signed communication to Banfield Machine Tools: 'We hereby offer to sell you 500 diamond core drill bits, priced at $300 each. This is a firm offer.' This offer was received by Banfield Machine Tools on March 7. On March 8, Triton mailed a notice to Banfield stating: Please be advised that our offer of March 5 is revoked. This notice of revocation was received by Banfield on March 10. On March 11, Banfield wired Triton: 'Consider you bound by your offer. Please enter our order as per your offer for 10 diamond core drill bits. Box two per case, hardwood casing only. Our draft for $3,000 has been sent under separate cover.' If the court were to conclude that a contract came into existence between Triton and Banfield on the strength of Triton's letter of March 5, and Banfield's reply of March 11, its terms would include: (A) (B) (C)(D)

    • A.

      Only those terms set forth in Triton's letter of March 5, because the offeror has not assented to any enlargement of the terms regarding mode of shipment.

    • B.

      All terms set forth in Triton's offer plus consistent additional terms proposed in Banfield's acceptance.

    • C.

      All terms set forth in Triton's offer plus those in Banfield's attempted acceptance, provided that Banfield's proposals did not amount to a material alteration of Triton's offer

    • D.

      The terms of the offer plus all those in the purported acceptance which did not amount to a material alteration of the offer and to which the offeror did not object within a reasonable time.

    Correct Answer
    D. The terms of the offer plus all those in the purported acceptance which did not amount to a material alteration of the offer and to which the offeror did not object within a reasonable time.
    Explanation
    (D) properly states the U.C.C. position regarding the terms of the contract. Under the U.C.C., if both parties to a contract are merchants, additional terms in an acceptance will be included in the contract unless
    (i) they materially alter the original contract;
    (ii) the offer expressly limits acceptance to the terms of the offer; or
    (iii) the offeror has already objected to the particular terms, or objects within a reasonable time after notice of them is received. [U.C.C. |2- 207(2)] Triton and Banfield are both merchants because they regularly deal in goods such as the drill bits. [U.C.C. |2-104(1)] Therefore, their contract for the sale of the bits comes under Article 2 and will include the term of Triton's offer plus those in Banfield's purported acceptance which do not amount to a material alteration of the offer or to which Triton did not object within a reasonable time. Note that Triton's offer did not expressly limit acceptance of its terms.

    (A) is incorrect because this was a contract between two merchants. Contract formation under the U.C.C. for contracts between merchants is governed by the rule stated above. If one of the parties were not a merchant,

    (A) would be correct|if one of the parties to a contract for sale of goods is not a merchant and the acceptance includes additional or different terms, such terms are considered to be mere proposals that do not become part of the contract unless the offeror accepts. [U.C.C. |2-207(1)] However, because this is a contract between merchants, Banfield's terms regarding mode of shipment will be included in the contract unless they materially alter the original contract, as discussed above.

    (B) is incorrect because it does not fully state the Code's 'battle of the forms' provision. The choice fails to mention Triton's power to object within a reasonable time.

    (C) is incorrect because, like

    (B), it does not note Triton's power to object within a reasonable time.

    Rate this question:

  • 47. 

    On April 1, Stan, a law student, agreed to sell his prized possession|an antique dictionary| to Betsy for $1,500, so that he could pay for his bar review course. Because Stan did not have to pay for his course until April 30, the written contract between Stan and Betsy provided that the dictionary would not be delivered to Betsy until April 20, thus giving Stan a last few extra days to peruse the dictionary's pages. Late on April 15, a terrible fire swept through Stan's apartment building and the dictionary was destroyed. Firefighters determined that the blaze started when Hugh, the tenant living below Stan, fell asleep while doing his taxes and dropped his cigarette onto his paperwork. Fortunately for Stan, he had insurance that covered all of his damages, including compensation for the destroyed dictionary. On April 20, Stan told Betsy of the fire, but still demanded payment, claiming that Betsy was the equitable owner of the dictionary when it was destroyed, and explaining that she could have obtained insurance on the dictionary had she wanted to, since she had an insurable interest in the dictionary as soon as the contract was made. Betsy refused to pay. Stan brings an action against Betsy for the $1,500. Who will prevail?

    • A.

      Betsy, because Stan was fully compensated for his dictionary and making Betsy pay would therefore result in unjust enrichment.

    • B.

      Betsy, because destruction of the dictionary avoids the contract and discharges her duty to pay.

    • C.

      Stan, because when he contracted with Betsy, the risk of loss passed to her.

    • D.

      Stan, because of the doctrine of equitable conversion.

    Correct Answer
    B. Betsy, because destruction of the dictionary avoids the contract and discharges her duty to pay.
    Explanation
    (B) Betsy will prevail because complete destruction of the dictionary results in avoidance of the contract and discharge of her duty to pay, since Stan still had the risk of loss. Since the contract here is for the sale of goods, it is governed by the Uniform Commercial Code
    ('U.C.C.'). Under the U.C.C., if a contract requires for its performance particular goods identified when the contract is made, and, before risk of loss passes to the buyer, the goods are destroyed without the fault of either party, the contract is avoided. [U.C.C. |2-613] All of the elements of section 2-613 are present here. The contract required Stan's particular dictionary, which was identified at the time the contract was made. The risk of loss had not yet passed to Betsy because in a sale by a nonmerchant such as Stan, risk of loss does not pass to the buyer until tender [U.C.C. |2-509], and Stan never tendered the dictionary here
    (there was no actual tender and delivery was not due until April 20). Finally, the fire was caused by Hugh's careless smoking, and so the goods were destroyed without the fault of either party. Thus, the contract is avoided.
    (The same conclusion would result under the common law doctrine of impossibility|all executory duties are discharged when the subsequent destruction of the subject matter of a contract renders performance impossible.)

    (A) is incorrect because the U.C.C. contains no such rule. The only U.C.C. remedy that depends on an injured party's insurance involves the risk of loss after the buyer's revocation of acceptance or wrongful repudiation under section 2-510. Here, Betsy does not have to pay because the destruction of the dictionary discharged her duty to do so.

    (C) is wrong because, as explained above, the risk of loss had not yet passed to Betsy.

    (D) is wrong because the U.C.C. does not follow the doctrine of equitable conversion; rather, the Code contains very specific risk of loss rules, as detailed above.

    Rate this question:

  • 48. 

    Retailer entered into a written contract with Wholesaler whereby Wholesaler agreed to sell and Retailer agreed to buy 100 boxes of 'Red Star' sunglasses, manufactured by Redblok Corporation. The agreed-upon price was $75 per box. Two weeks before the specified delivery date, Wholesaler notified Retailer, 'Because of unexpected high demand for sunglasses this season, we will be unable to fill your order.' Although Retailer learned that the needed quantity of 'Red Star' sunglasses could be shipped within two days for $83 per box from a supplier in another area, Retailer instead purchased 100 boxes of 'Red Seal' sunglasses locally at a cost of $90 per box. The 'Red Seal' glasses were also manufactured by Redblok Corporation but were of a slightly higher quality than the 'Red Star' model. A few days before the original delivery date, Wholesaler notified Retailer that it would fill the order, and tendered 100 boxes of 'Red Star' sunglasses on the date of delivery. However, Retailer refused to accept them. At that time, the wholesale market price of 'Red Star' sunglasses had declined to $80 per box. If Retailer sues Wholesaler for damages based on Wholesaler's alleged breach, Retailer is likely to recover:

    • A.

      $1,500, representing the difference between the cost of cover and the contract price.

    • B.

      ) $800, representing the difference between the contract price and the nonlocal supplier's price.

    • C.

      $500, representing the difference between the contract price and the wholesale market price at the time of performance.

    • D.

      Nothing, because Retailer obtained cover without waiting a commercially reasonable time for Wholesaler to retract the repudiation.

    Correct Answer
    B. ) $800, representing the difference between the contract price and the nonlocal supplier's price.
    Explanation
    (B) Retailer is entitled to recover $800. Wholesaler's notice that it would be unable to fill Retailer's order constituted an anticipatory repudiation, which Retailer was entitled to treat as a total breach. Under the U.C.C., the buyer's basic remedy where the seller breaches by refusing to deliver is the difference between the contract price and either the market price or the cost of buying replacement goods
    ('cover'). If the buyer intends to fix damages based on the latter measure, the buyer must make a reasonable contract for substitute goods in good faith and without unreasonable delay. Here, Retailer chose to make a contract for a higher quality of sunglasses at a higher price, even though the model that he had originally ordered was available from a supplier outside the area. While Retailer need not find the lowest available price in the country or make a contract for substitute goods with an unreliable supplier, he was aware that he could have obtained the 'Red Star' sunglasses in plenty of time from the nonlocal supplier. Absent additional facts that would justify Retailer's decision, he can recover only the difference between the contract cost and a reasonable contract for substitute goods. Hence,

    (A) is wrong because Retailer's contract for cover probably would not be deemed to be commercially reasonable.
    (C) is wrong because Retailer's remedy based on market price would be determined at the time Retailer learned of the breach, not necessarily the time of performance. In the case of an anticipatory repudiation such as this, the buyer may either treat the anticipatory repudiation as a total breach and pursue his breach of contract remedies, or suspend his performance and await seller's performance for a commercially reasonable time. Retailer chose to treat Wholesaler's notice as a total repudiation and breach of contract. Hence, the market price remedy would be measured at that time because that is when Retailer 'learned of the breach,' rather than at the time of performance.

    (D) is wrong because the nonrepudiating party need not wait for the repudiating party to retract its repudiation. Retailer exercised its option to treat the repudiation as a total breach and buy substitute goods. Once that occurred, Wholesaler was not entitled to retract its repudiation and force Retailer to accept the sunglasses.

    Rate this question:

  • 49. 

    Ann leased commercial property to Brenda for a period of 10 years. The lease contained the following provision: 'No subleasing or assignment will be permitted unless with the written consent of the lessor.' One year later, Brenda assigned all interest in the lease to Carolyn, who assumed and agreed to perform the lessee's obligations under the terms of the lease. Ann learned of the assignment and wrote to Brenda that she had no objection to the assignment to Carolyn and agreed to accept rent from Carolyn instead of Brenda. Thereafter, Carolyn paid rent to Ann for a period of five years. Carolyn then defaulted and went into bankruptcy. In an appropriate action, Ann sued Brenda for rent due. If Ann loses, it will be because there was:

    • A.

      Laches.

    • B.

      An accord and satisfaction.

    • C.

      A novation.

    • D.

      An attornment.

    Correct Answer
    C. A novation.
    Explanation
    C EXPLANATION
    (C) If Ann loses it will be because there was a novation. A novation occurs when a new contract substitutes a new party to receive benefits and assume duties that had belonged to one of the original parties under the terms of an old contract. All the parties must agree to the substitution. A novation discharges the old contract. The elements of a valid novation are:
    (i) a previous valid contract;
    (ii) an agreement among all parties, including the new party to the new contract;
    (iii) the immediate extinguishment of contractual duties as between the original parties; and
    (iv) a valid and enforceable new contract. When lessor Ann wrote to Carolyn agreeing to accept rent and Carolyn began to do so, a new contract was formed, and Ann's rights against Brenda were extinguished.

    (A) is incorrect because laches is not available here. Laches is an equitable defense available when a party delays in bringing an equitable suit and the delay prejudices the defendant. The action here
    (for back rent) is at law, not equity. Moreover, nothing in the facts indicates that Ann delayed in bringing her action. Thus, laches is not available.

    (B) is incorrect because there was no accord and satisfaction. An accord occurs when the parties to a contract agree to substitute a different duty for an existing duty under the contract. Satisfaction occurs when the new duty is performed. There was no new duty here; rather, the parties agreed to substitute a new party under an existing contract
    (a novation).

    (D) is incorrect because attornment arises when the tenant agrees to accept a new landlord, and here Ann remained the landlord at all times.

    ________________________________________C EXPLANATION
    (C) If Ann loses it will be because there was a novation. A novation occurs when a new contract substitutes a new party to receive benefits and assume duties that had belonged to one of the original parties under the terms of an old contract. All the parties must agree to the substitution. A novation discharges the old contract. The elements of a valid novation are:
    (i) a previous valid contract;
    (ii) an agreement among all parties, including the new party to the new contract;
    (iii) the immediate extinguishment of contractual duties as between the original parties; and
    (iv) a valid and enforceable new contract. When lessor Ann wrote to Carolyn agreeing to accept rent and Carolyn began to do so, a new contract was formed, and Ann's rights against Brenda were extinguished.

    (A) is incorrect because laches is not available here. Laches is an equitable defense available when a party delays in bringing an equitable suit and the delay prejudices the defendant. The action here
    (for back rent) is at law, not equity. Moreover, nothing in the facts indicates that Ann delayed in bringing her action. Thus, laches is not available.

    (B) is incorrect because there was no accord and satisfaction. An accord occurs when the parties to a contract agree to substitute a different duty for an existing duty under the contract. Satisfaction occurs when the new duty is performed. There was no new duty here; rather, the parties agreed to substitute a new party under an existing contract
    (a novation).

    (D) is incorrect because attornment arises when the tenant agrees to accept a new landlord, and here Ann remained the landlord at all times.

    Rate this question:

  • 50. 

    Indegas wanted to ensure that it had sufficient stocks of gasoline and oil for its independent dealerships; it therefore entered into contracts with various refineries to purchase the gasoline they produced. One such agreement was with Cal-Tex, whereby Indegas was given the right to purchase all gasoline refined by Cal-Tex for the next five years at a price set at 95% of the domestic market price at the time of delivery. Indegas agreed to purchase no less than 5,000 gallons a week and to use its own tankers to transport the gasoline from Cal-Tex's refinery to its storage facilities. At the time this contract was signed, Indegas gave written notice to Cal- Tex that it intended to buy all gasoline produced by Cal-Tex until further notice. For the first year, Indegas continued to purchase all gasoline produced by Cal-Tex. However, at the end of that year Cal-Tex purchased an out-of-state refinery that refined all its gasoline from domestic shale oil. As a result of this merger and of more efficient plant methods, Cal- Tex doubled its production of gasoline by the next year. At a meeting between Cal-Tex and Indegas, Cal-Tex's president noted that Indegas was getting as much gasoline as it needed, and that Cal-Tex intended to sell the extra gasoline it was producing on foreign markets at a higher price than Indegas was paying. Indegas agreed with this proposal and signed an addendum to the original agreement reflecting this change. Thereafter, Indegas continued to purchase one-half Cal-Tex's total capacity until the following year when, as a result of a foreign oil boycott, Cal-Tex was unable to purchase sufficient foreign oil to operate profitably its local refinery. Cal-Tex, by letter, notified Indegas that it could no longer deliver gasoline to it in accordance with their agreement. Nevertheless, Cal-Tex's other refinery was able to increase its production by 25% and continue to sell to foreign markets. Assuming that Indegas had an enforceable agreement, the modification of this contract made by the addendum was:

    • A.

      Enforceable to the extent that Indegas purchased the gasoline.

    • B.

      Enforceable in all respects.

    • C.

      Unenforceable, because there was no consideration for Indegas's agreement to take only one-half of the gasoline produced.

    • D.

      Unenforceable, because the contract did not state the amount of gasoline produced by each refinery.

    Correct Answer
    B. Enforceable in all respects.
    Explanation
    (B) Although there was no consideration for Indegas to take only one-half of the bargained-for production, none is required because this is a contract for the sale of goods, and subject to U.C.C. section 2-609
    (which provides that good-faith modification is enforceable regardless of lack of consideration).

    (C) is therefore wrong.

    (A) and

    (D) make no sense.

    Rate this question:

Quiz Review Timeline +

Our quizzes are rigorously reviewed, monitored and continuously updated by our expert board to maintain accuracy, relevance, and timeliness.

  • Current Version
  • May 17, 2023
    Quiz Edited by
    ProProfs Editorial Team
  • May 08, 2010
    Quiz Created by
    Dmatski
Back to Top Back to top
Advertisement
×

Wait!
Here's an interesting quiz for you.

We have other quizzes matching your interest.